You are on page 1of 141

1

A 47-year-old barmaid presents to the Accident and Emergency department


with a 12-hour history of right upper quadrant pain and vomiting. She says
that the pain is radiating to her right scapula and exacerbates on breathing.
She appears pale but not jaundiced. On examination, her pulse rate is
98/min, blood pressure is 126/84 mmHg and temperature is 37.6 C.
Abdominal examination reveals tenderness over the right hypochondrium
but no mass is palpable. Plain radiographs of the abdomen (supine) and
chest (erect) are unremarkable.

From the options below choose the ONE that you think is the most likely
diagnosis in this patient:
Single best answer question choose ONE true option only.

Perforated peptic ulcer

Acute pancreatitis

Acute biliary cholangitis

Acute cholecystitis YOUR ANSWER

Infective hepatitis

YOUR ANSWER WAS CORRECT

The Answer
Comment on this Question

The history, signs and symptoms in this patient are suggestive of acute
cholecystitis. Acute cholecystitis is more common in females over the age of
40 and with high BMI. Gallstones are the commonest cause for acute
cholecystitis. Obstruction of the common bile duct due to stones leads to
accumulation of bile andinflammation, resulting in an acutely inflammed
gall bladder. Other risk factors for acute cholecystitis include alcohol abuse
and tumours of the gall bladder. The signs and symptoms of acute
cholecystitis include severe right hypochondrial pain exacerbated by
respiration, nausea and vomiting, and increase in temperature. The rise in
temperature is frequently mild to moderate; a very high temperature with or
without chills and rigors may point to a diagnosis of acute cholangitis. A
tender, inflamed gall bladder may be palpable in some patients. Likewise,
jaundice may or may not be present. The differential diagnoses for acute
cholecystitisinclude acute pancreatitis, peptic ulcer disease or perforated
peptic ulcer, appendicitis, acute infective hepatitis and pleurisy.

You are reviewing a patient in the plastic surgery clinic who had a malignant
melanoma excised.

Which one of the following is suggestive of a good prognosis in malignant


melanoma?

Select one answer only

Low Breslow thickness YOUR ANSWER


Male sex

Mucosal primary site

Older age

The presence of ulceration

YOUR ANSWER WAS CORRECT

The Answer

Comment on this Question

Male sex, increased tumour thickness, the presence of ulceration, older age
and mucosal involvement are predictors of a poor outcome in melanoma.
The prognosis also depends on the lymph node involvement and growth
pattern. The 5-year survival rate is 90% for stage I, 50% for stage II, 30% for
stage III and < 1% for stage IV disease. While the so-called Celtic type
races appear more susceptible to the development of melanoma, non-white
populations with melanoma have a worse overall outcome.

Theme: Polyps

A Inflammatory polyps

B Metaplastic polyps
C PeutzJeghers polyps

D Tubular adenomatous polyps

E Villous adenomatous polyps

For each of the descriptions below, select the most appropriate polyp from the
above list. Each option may be used once, more than once, or not at all.

Scenario 1

Are usually a consequence of a severe episode of ulcerative colitis

A - Inflammatory polyps CORRECT ANSWER

A Inflammatory polyps

YOUR ANSWER WAS CORRECT

Scenario 2

Are hamartomas

C - PeutzJeghers polyps CORRECT ANSWER

C PeutzJeghers polyps

YOUR ANSWER WAS CORRECT

Scenario 3

May cause hypokalaemia

D - Tubular adenomatous polyps YOUR ANSWER

E - Villous adenomatous polyps CORRECT ANSWER

E Villous adenomatous polyps


YOUR ANSWER WAS INCORRECT

Scenario 4

Have the greatest malignant potential of all colonic polyps

E - Villous adenomatous polyps CORRECT ANSWER

E Villous adenomatous polyps

YOUR ANSWER WAS CORRECT

Scenario 5

Are the commonest type of polyp seen in familial adenomatous polyposis

B - Metaplastic polyps YOUR ANSWER

D - Tubular adenomatous polyps CORRECT ANSWER

D Tubular adenomatous polyps

YOUR ANSWER WAS INCORRECT

The Answer

Comment on this Question


Villous adenomas of the rectum frequently present with the passage of bright-
red blood and mucus per rectum. If villous adenomas are large, mucus secretion
is significant and hypokalaemia may occur. Villous adenomas have the greatest
malignant potential of all colorectal polyps. PeutzJeghers polyps have no
malignant potential themselves, but are associated with an increased risk of
other malignant gut polyps. Metaplastic polyps are also known as hyperplastic
polyps. They are symptomless, occurring throughout the large bowel and are the
most common type of polyp found in the rectum. They are thought not to
undergo dysplastic or neoplastic change. Inflammatory polyps have no
malignant potential.

A 56-year-old man presents with episodes of severe epigastric pain radiating


through to his back for several weeks. He also reports he has noticed his
stools tend to float more recently and are very difficult to flush away. On
examination erythema abigne is noted over the epigastrium.

What is the most likely diagnosis?

Select one answer only

Acute pancreatitis

Biliary colic
Coeliac disease

Chronic pancreatitis YOUR ANSWER

Pancreatic Cancer

YOUR ANSWER WAS CORRECT

The Answer

Comment on this Question

There are a number of features of this case to suggest it is chronic pancreatitis


rather than acute- its prolonged history and the patient has symptoms
suggestive of steatorrhoea due to an inability to absorb fats due to reduced
pancreatic enzyme production. Erythema abigne is caused by using a hot water
bottle to relieve pain and is often seen in the epigastrium or on the back.

Theme: Venous drainage of the suprarenal gland

A Splenic vein

B Left renal vein

C Right renal vein

D Inferior mesenteric vein


E Inferior vena cava

Pick the most appropriate option from the above list. Each option may be used
once only, more than once or not at all.

Scenario 1

What is the venous drainage for the right suprarenal gland?

E - Inferior vena cava CORRECT ANSWER

E - Inferior vena cava

YOUR ANSWER WAS CORRECT

Scenario 2

What is the venous drainage for the left suprarenal gland?

B - Left renal vein CORRECT ANSWER

B - Left renal vein

YOUR ANSWER WAS CORRECT

The Answer

Comment on this Question

The right suprarenal vein enters the inferior vena cava and the left vein is longer
and enters the left renal vein.

6
Theme: Groin lumps

A Ectopic testis

B Femoral artery aneurysm

C Femoral hernia

D Hydrocoele of the spermatic cord

E Inguinal hernia

F Inguinal lymphadenopathy

G Lipoma

H Pseudo-aneurysm

I Psoas abscess

J Psoas bursa

K Saphena varix

L Sarcoma

The following patients all present with a lump in the groin. For each scenario please select
the most appropriate diagnosis from the above list. The items may be used once, more
than once, or not at all.

Scenario 1
A 36-year-old Asian immigrant presents to The Emergency Department with a tender,
fluctuant mass in his left femoral triangle. He gives a history of night sweats, weight
loss and a painful left hip.

I - Psoas abscess CORRECT ANSWER

I Psoas abscess

Although this mass could be attributed to lymphadenopathy (probably


secondary to his clear history of tuberculosis), the fluctuant nature and the
presence of ipsilateral hip pain point more readily to symptoms of a psoas
abscess. Psoas abscesses develop either from infection of unknown origin or as
a consequence of infection spreading from an adjacent organ. The risk factors
for primary psoas abscess are not known; however, trauma to the muscle may
be an important factor in 1820% of cases. Low socioeconomic class and poor
nutrition have also been cited as possible predisposing factors. A major risk
factor for secondary psoas abscess is gastrointestinal pathology (inflammatory
bowel disease, appendicitis, diverticulitis, bowel cancer and Crohns disease),
and the source is a gastrointestinal infection in 80% of individuals. Prior to
modern anti-tuberculous therapy, psoas abscesses occurred in up to 20% of
patients with spinal tuberculosis. Treatment is now usually (initially at least) by
percutaneous drainage under ultrasound or computed tomography guidance.

YOUR ANSWER WAS CORRECT

Scenario 2

A 68-year-old man attends Casualty suffering from drowsiness and confusion. His wife
reports a 12-h history of vomiting and abdominal pain. On examination he is clearly
dehydrated, his abdomen is distended and he has high-pitched bowel sounds. More
detailed assessment reveals a small painful swelling in his right groin crease.

C - Femoral hernia CORRECT ANSWER


C Femoral hernia

This gentleman has signs of small bowel obstruction secondary to a


strangulated femoral hernia. A small complicated hernia present in the groin
crease in the elderly with no prior history of a reducible lump is much more likely
to be a femoral than an inguinal hernia (although inguinal hernias are
approximately ten times more common in general).

YOUR ANSWER WAS CORRECT

Scenario 3

A 62-year-old claudicant returns to the ward from the vascular assessment laboratory,
following an angiogram of his lower limbs. The nurse is concerned about a swelling in
his left groin. On closer examination you note a firm mass with a transmissible pulse.

H - Pseudo-aneurysm CORRECT ANSWER

H Pseudo-aneurysm

The nature of this mans investigation points to the diagnosis of a


pseudoaneurysm. Failure to compress the site of arterial cannulation (a
traumatic breach to the vessel wall) leads to extravasation of arterial blood. A
haematoma then forms in the soft tissues around the artery, which produces a
transmissible pulse. A true aneurysm is one that involves all the layers of the
arterial wall (three layers) and is described as being expansile.

YOUR ANSWER WAS CORRECT

7
Theme: Intestinal obstruction

A Adhesions

B Bezoar

C Colonic carcinoma

D Crohns disease

E Postoperative ileus

F Diverticular disease

G Extra-luminal neoplastic compression

H Gallstone ileus

I Hernia

J Intussusception

K Intestinal pseudo-obstruction

L Small bowel tumour

M Volvulus

The following scenarios describe patients with signs of obstruction. From the above list
choose the most likely cause. Each item may be used once, more than once, or not at all.

Scenario 1

A 75-year-old woman presents with an 8-h history of vomiting and colicky central
abdominal pain. Her abdomen appears mildly distended and although she is
uncomfortable, there is no obvious tenderness. A plain abdominal radiograph
demonstrates pneumobilia.

K - Intestinal pseudo-obstruction YOUR ANSWER

H - Gallstone ileus CORRECT ANSWER

H Gallstone ileus

Gallstones are responsible for less than 1% of all cases of small bowel
obstruction. About 90% of stones entering the intestine will impact in the
terminal ileum, although other reported sites include the duodenum, jejunum,
colon and rectum. A stone formed in the gall bladder enters the small bowel via a
biliary-enteric fistula, usually between the gallbladder and duodenum. Patients
tend to be elderly and often do not report a history of cholecystitis. On X-ray the
presence of a pneumobilia is pathognomic, provided there is no recent history of
biliary-intestinal bypass or sphincterotomy.

YOUR ANSWER WAS INCORRECT

Scenario 2

An 88-year-old man is admitted from a nursing home with confusion, lower abdominal
pain and gross distension. A plain abdominal radiograph demonstrates the coffee-bean
sign.

M - Volvulus CORRECT ANSWER

M Volvulus

The case gives the classical history of a patient with a sigmoid volvulus who is
typically elderly and institutionalised, suffering with chronic constipation and
regular laxative use. There is usually a history of similar episodes in the past,
which resolve following untwisting of the sigmoid mesentery. Typical X-ray
findings are of a large oval gas shadow on the left hand side, which is looped
onto itself, the so-called bent inner tube or coffee bean sign. Treatment is by
decompression by means of a flatus tube inserted via a sigmoidoscope, done of
course at arms length! Should this fail then operative untwisting with or
without resection may be required.

YOUR ANSWER WAS CORRECT

Scenario 3

A 30-year-old man presents with a several month history of rightsided abdominal pain
and some diarrhoea and weight loss. Subsequent radiological investigations
demonstrate the string sign of Kantor.

D - Crohns disease CORRECT ANSWER

D Crohns disease

The history is fairly typical of that for ileocaecal Crohns disease (the
commonest site: 50%). Disease of the terminal ileum may result in fine string-
like-calibre strictures in the right iliac fossa on barium studies. This is known as
the string sign of Kantor.

YOUR ANSWER WAS CORRECT

Scenario 4

A 78-year-old man with known small cell carcinoma of the lung attends with abdominal
pain, distension and vomiting. The abdomen is soft and non-tender with no masses. He
has had no previous abdominal surgery and there is no hernia.

L - Small bowel tumour YOUR ANSWER

K - Intestinal pseudo-obstruction CORRECT ANSWER


K Intestinal pseudo-obstruction

This is a bit of an honours question it is one of the rarer paraneoplastic


syndromes associated with small cell carcinoma. It is caused by an autoimmune
inflammation/destruction of the myenteric plexus.

YOUR ANSWER WAS INCORRECT

A 35-year-old male with known Ulcerative colitis (UC) reports a generalised


itching, fatigue and abdominal pain. On examination, he is found to be jaundiced
and his blood tests show a markedly raised bilirubin and alkaline phosphatase.
An MRCP shows multiple strictures in the biliary tree.

What is the most likely diagnosis?

Select one answer only

Cholangitis

Cholecystitis

Primary biliary cirrhosis

Primary sclerosing cholangitis YOUR ANSWER


Wilsons disease

YOUR ANSWER WAS CORRECT

The Answer

Comment on this Question

Primary sclerosing cholangitis (PSC) is a condition in which inflammation,


fibrosis and strictures of the intra- and extra-hepatic bile ducts occur. MRCP
shows multiple strictures in the biliary tree and a characteristic beaded
appearance. Around 80% of patients with PSC will have a diagnosis of UC.

A 57-year-old businessman from Taiwan presents to his GP with symptoms of


malaise, weight loss and right upper quadrant pain. On examination he is noted
to have hepatomegaly which is irregular. A bruit is also heard over the liver. His
blood tests show a very raised alpha-fetoprotein.

What is the most likely diagnosis?

Select one answer only

Cholangiocarcinoma

Fibroarcoma of the liver


Gallstones

Hepatocellular carcinoma (HCC) CORRECT ANSWER

Liver haemangioma

The Answer

Hepatocellular carcinoma (HCC) is the commonest malignant tumour of


hepatocytes accounting for 90% of primary liver cancers. It is more common in
China and Taiwan and is caused by chronic Hepatitis B or C infection, or
cirrhosis. On examination they may be palpable as an irregular hepatomegaly
and sometimes a bruit may be heard over them. The alpa-fetoprotein levels are
raised in 80% of patients.

10

A 17-year-old A-level student presents with diarrhoea, abdominal pain and


weight loss. On examination, she is tender in the right iliac fossa (RIF) and on
further questioning she had a perianal abscess drained 1 year ago. A Barium
enema shows cobblestoning in the RIF.

What is the most likely diagnosis?

Select one answer only


Coeliac disease

Crohns disease YOUR ANSWER

Irritable bowel syndrome

Tropical sprue

Ulcerative colitis

YOUR ANSWER WAS CORRECT

The Answer

Comment on this Question

Crohns disease is a form of inflammatory bowel disease that has several


features in common with Ulcerative colitis. However, it is characterised by
transmural granulomatous inflammation and can affect any part of the gut. It is
most commonly found in the terminal ileum (and therefore may produce
discomfort in the RIF). Other features seen include diarrhoea and weight loss.
Perianal disease is also common and may manifest as abscesses and fistulae.
Barium enemas will classically show cobblestoning and rose-thorn ulcers. MRI
can also be used to assess the extent of pelvic disease.

11
Theme: Investigations for abdominal pain

A CT abdomen

B ECG

C Erect chest X-ray

D FBC

E Mesenteric angiography

F Supine abdominal X-ray

G Urea

For each of the patients below, select the most likely specific diagnostic investigation from
the above list. Each option may be used once, more than once, or not at all.

Scenario 1

An 18-year-old man presents with 24 hours of generalised abdominal pain, which has
now shifted to the right iliac fossa. Examination reveals a soft abdomen with
tenderness in the right fossa and no evidence of peritonism. The patient is
haemodynamically stable.

A - CT abdomen CORRECT ANSWER

A CT abdomen

The most likely diagnosis for the first patient is appendicitis. FBC measurement
is useful in lending support to the diagnosis, in that it may show a neutrophilia,
but a normal reading should not affect management. CT of the abdomen is the
most specific investigation of the options offered in this question.

YOUR ANSWER WAS CORRECT


Scenario 2

A 59-year-old woman with a history of rheumatoid arthritis treated with NSAIDs, gold
and steroids, presents with 4 hours of acute abdominal pain. This was made worse by
the ambulance ride. Her abdomen is generally peritonitic on examination.

C - Erect chest X-ray CORRECT ANSWER

C Erect chest X-ray

Patient 2 has a good history for perforation of a viscus, probably a peptic ulcer
she is on several ulcer-promoting medications. An erect chest X-ray (CXR)
should be performed after the patient has been upright for several minutes to
look for free gas under the diaphragm.

YOUR ANSWER WAS CORRECT

Scenario 3

An 85-year-old woman presents with profuse, fresh, red rectal bleeding. She is
hypotensive, with a fast irregular pulse.

E - Mesenteric angiography CORRECT ANSWER

E Mesenteric angiography

While an FBC is important in patient 3 it will not lead to diagnosing the cause of
the problem. A mesenteric angiogram is useful if the patient is bleeding at a rate
of > 1 ml/min.

YOUR ANSWER WAS CORRECT

Scenario 4
A 62-year-old woman is admitted with vomiting, colicky abdominal pain and a
distended abdomen. She has previously undergone multiple gynaecological
operations.

A - CT abdomen CORRECT ANSWER

A - CT abdomen

Patient 4 may be developing acute intestinal obstruction secondary to adhesions


from previous abdominal surgery. Although plain abdominal radiography may
demonstrate distended bowel loops, CT of the abdomen and pelvis should reveal
the level and likely cause of the obstruction, thereby guiding the operating
surgeon.

YOUR ANSWER WAS CORRECT

Comment on this Question

12

Theme: Common abdominal operations

A Abdomino-perineal excision of rectum

B Anterior resection of rectum

C Completion colectomy

D Hartmanns procedure

E Laparoscopic cholecystectomy
F Loop colostomy

G Left hemicolectomy

H On-table lavage, primary resection and anastomosis

I Restorative proctocolectomy

J Ileocolic resection

K Stricturoplasty

L Total mesorectal excision

The following patients are all to undergo surgery. Please select the most
appropriate operation from the above list. The items may be used once, more
than once, or not at all.

Scenario 1

You have seen a 74-year-old man in The Emergency Department. He gives a 3-day
history of localised lower abdominal pain, which has become generalised over the past
24 h. He is known to suffer with ischaemic heart disease and sigmoid diverticulosis. On
examination he is dehydrated, tachycardic (pulse 120/min), pale, sweating and pyrexial
(38C). Abdominal examination reveals the presence of diffuse abdominal tenderness,
guarding and rigidity. The results of investigations performed so far have demonstrated
a raised white cell count of 22.5 x 109/litre, acidosis on arterial blood gases and
pneumoperitoneum on erect chest X-ray.

D - Hartmanns procedure CORRECT ANSWER

D Hartmanns procedure

The clinical picture is that of a perforated diverticulum with faecal peritonitis. In


an elderly patient with gross faecal peritonitis the mortality rate is over 50% and
in such circumstances a lengthy procedure is not indicated. In selected cases
with a small leak, minimal soiling and a fit patient, primary resection with on-
table lavage and anastomosis can be performed. However, this is not indicated
in the case described. Hartmanns procedure is also used in an emergency for
obstructing conditions affecting the sigmoid region of the colon (especially in
high-risk patients). The diseased or obstructed area is excised but an
anastomosis of the bowel is not undertaken for reasons of safety at the time, and
the high risk of anastomotic dehiscence post-operatively. The proximal colon is
therefore brought to the skin as a stoma and the rectal stump is oversewn. The
colostomy may be reversed at a later date, but this in itself is no small
undertaking.

YOUR ANSWER WAS CORRECT

Scenario 2

A 23-year-old man with long-standing Crohns disease is seen in the combined colitis
clinic. He gives you a long history from adolescence of recurrent episodes of colicky
abdominal pain and vomiting. A recent colonoscopy has not demonstrated any
significant colonic pathology and, in addition, he has had a barium meal and follow-
through, which has been reported as demonstrating significant terminal ileal stricturing.
He continues to be symptomatic and is now keen on having surgery.

K - Stricturoplasty YOUR ANSWER

J - Ileocolic resection CORRECT ANSWER

J Ileocolic resection
Surgical resection will not cure Crohns disease and is usually performed for
complications. The overall strategy is to be as conservative as possible to
preserve functional gut length. Indications for surgery include recurrent
intestinal obstruction, intestinal fistulae, fulminant colitis, malignant change and
peri-anal disease. The whole of the gastrointestinal tract should be examined
prior to undertaking any resection either pre-operatively or at laparotomy.
Proximal small bowel strictures can be treated with segmental resection if only
isolated areas are affected, or alternatively, with stricturoplasty of multiple
involved segments. A right hemicolectomy involves taking the ileocolic, right
colic and right branch of middle colic vessels resulting in loss of more bowel
than for an ileocolic resection. Here only the Ileocolic vessels are taken. For first
presentation Crohns, the best treatment option is an Ileocolic resection. For
fibrous Crohns strictures, strictureplasty would be a good option.

YOUR ANSWER WAS INCORRECT

Scenario 3

A 64-year-old woman has been seen in the rectal bleed clinic and a diagnosis of
a low rectal cancer (7 cm from the anal verge) has been made. A magnetic
resonance imaging scan demonstrates a resectable lesion.

A - Abdomino-perineal excision of rectum YOUR ANSWER

B - Anterior resection of rectum CORRECT ANSWER

B Anterior resection of rectum


This is the operation of choice for middle and lower third tumours of the rectum
that are amenable to resection without recourse to abdominoperineal excision of
rectum (APER). The operation, championed by Bill Heald, attends to the main
cause of local failure (recurrence) of cancer by addressing both the
circumferential resection margins as well as the distal resection margin. APER is
used when an adequate distal resection margin is not achievable by abdominal
approach usually because the tumour has invaded the sphincters.

YOUR ANSWER WAS INCORRECT

Reference:

http://emedicine.medscape.com/article/281237-treatment

13

Theme: Gastrointestinal (GI) investigations

A Colonoscopy

B Selective mesenteric angiography

C Colloid red cell scan


D Oesophagogastroduodenoscopy

E Computerised tomography

F Magnetic resonance imaging (MRI) scan

Select the most appropriate first investigation for the assessment of the patients
below. Each option may be used once, more than once or not at all.

Scenario 1

A 60-year-old man presenting with massive rectal bleeding. He has passed 800
ml of clotted blood with no evidence of melaena. There is no history of ingestion
of non-steroidal anti-inflammatory drugs (NSAIDs) or change in bowel habit.
Following aggressive resuscitation he now has a blood pressure (BP) of 120/65
mmHg, and a pulse of 86 beats/min. Rigid sigmoidoscopy is normal.

B - Selective mesenteric angiography YOUR ANSWER

D - Oesophagogastroduodenoscopy CORRECT ANSWER

In cases of massive bleeding per rectum, one should exclude an upper GI tract
haemorrhage first.

YOUR ANSWER WAS INCORRECT

Scenario 2

A 65-year-old woman presenting with a tender non-pulsatile mass in the left iliac
fossa. It is associated with a swinging pyrexia. There is mild abdominal
distension but no change in bowel habit.

E - Computerised tomography CORRECT ANSWER


Computerised tomography is the gold-standard investigation for diverticular
abscess.

YOUR ANSWER WAS CORRECT

Scenario 3

A 35-year-old man with long-standing Crohns disease presenting with complex


anal fistulae. Previous fistulograms have been unhelpful.

F - Magnetic resonance imaging (MRI) scan CORRECT ANSWER

MRI is now considered an excellent imaging modality for investigation of


complex anal fistulae.

YOUR ANSWER WAS CORRECT

14

Theme: The abdominal mass

A Abdominal aortic aneurysm

B Appendix mass

C Carcinoma of the caecum

D Carcinoma of the head of the pancreas

E Carcinoma of the sigmoid colon


F Carcinoma of the stomach

G Crohns mass

H Diverticular mass

I Empyema of the gallbladder

J Hepatomegaly

K Mesenteric cyst

L Mucocoele of the gallbladder

M Pancreatic pseudocyst

N Renal cell carcinoma

O Splenomegaly

The following patients have all presented with a palpable abdominal mass.
Please select the most appropriate diagnosis from the above list. The items may
be used once, more than once, or not at all.

Scenario 1

A 57-year-old woman presents with a 6-month history of a dull ache in the right iliac
fossa associated with anorexia and lethargy. The pain has become acute over the past
week with episodes of periumbilical colic and abdominal distension, which is relieved
on vomiting. On examination the patient is pale and dehydrated. Abdominal
examination reveals the presence of generalised abdominal distension and a firm,
irregular mass in the right iliac fossa.

G - Crohns mass YOUR ANSWER

C - Carcinoma of the caecum CORRECT ANSWER


C Carcinoma of the caecum

In contrast with left-sided tumours, caecal carcinoma has an insidious onset


with generalised features of malignant disease, such as anaemia, anorexia,
weight loss and lethargy. Late features include a palpable mass in the right iliac
fossa and obstruction to the ileocaecal valve resulting in small bowel
obstruction. Obstruction of the appendiceal orifice can give rise to symptoms
and signs of acute appendicitis, a fact that should be remembered when faced
with a patient over the age of 40 with appendicitis.

YOUR ANSWER WAS INCORRECT

Scenario 2

A 56-year-old woman presents with a 2-week history of increasing jaundice and


pruritis. Direct questioning reveals that over the past few months she has had some
vague epigastric pain, radiating to the back and has lost 2 stones in weight. On
examination a smooth hemi-ovoid mass is palpable in the right upper quadrant, which
moves with respiration. It is dull to percussion.

D - Carcinoma of the head of the pancreas CORRECT ANSWER

D Carcinoma of the head of the pancreas

The history and findings are of obstructive jaundice with a palpable gallbladder.
Courvoisiers law states when the gallbladder is palpable and the patient is
jaundiced, the obstruction of the bile duct causing the jaundice is unlikely to be
a stone because previous inflammation will have made the gallbladder thick and
non-distensible. While there are a few exceptions to this rule, the history of
substantial weight loss, and of pain radiating to the left side of the back strongly
indicate the likelihood of pancreatic carcinoma which is the commonest cause
of malignant biliary obstruction.
YOUR ANSWER WAS CORRECT

Scenario 3

An 18-year-old woman presents with a history of a painless, central abdominal


swelling. She is concerned, as this has gradually increased in size without causing any
symptoms. On examination she is anxious but otherwise appears well. Abdominal
examination reveals a fluctuant, spherical mass near the umbilicus. Of note, the mass
is mobile, frequently slipping towards the left iliac fossa during palpation.

K - Mesenteric cyst CORRECT ANSWER

K Mesenteric cyst

These cysts are found most commonly in the mesentery of the ileum arising
from congenitally misplaced lymphatic tissue (chylolymphatic cyst) or from
remnants of reduplicated bowel (enterogenous cyst). They typically present in
the second decade of life as a painless swelling or with recurrent episodes of
abdominal pain. Acute abdominal pain may arise following rupture or bleeding
into a cyst. On examination the characteristic finding is that of a fluctuant,
resonant, spherical swelling, close to the umbilicus. The cyst is freely mobile in
a plane at right angles to the root of the mesentery and may slip during the
course of the examination.

YOUR ANSWER WAS CORRECT

15

Theme: Gastrointestinal (GI) haemorrhage

A Upper gastrointestinal endoscopy


B Red blood cell scan

C Colonoscopy

D Selective angiography

E Double contrast barium enema

F Barium meal and follow through

For each of the patients described below, select the most likely single
investigation from the list of options above. Each option may be used once,
more than once or not at all.

Scenario 1

A 50-year-old man presents to the emergency department with dizziness and melaena.

A - Upper gastrointestinal endoscopy CORRECT ANSWER

Peptic ulceration is the commonest cause for such a presentation and therefore
upper gastrointestinal endoscopy is the investigation of choice in this patient.
The procedure may also allow treatment e.g. adrenaline injection.

Scenario 2

A 65-year-old woman presents with dark red rectal bleeding, hypotension (blood
pressure 95/60 mmHg), and tachycardia (pulse 100 beats/min). Full blood count
reveals anaemia (haemoglobin 8.6 g/dl). Upper GI endoscopy shows no
abnormality. Colonoscopy reveals dark red blood in the colon and no other
abnormality.

E - Double contrast barium enema YOUR ANSWER

D - Selective angiography CORRECT ANSWER

Selective angiography is a superior investigation in this instance. Although of


interest, the likely diagnosis in this case does not significantly affect the choice
of investigation. Mesenteric angiography is the logical next step in the
management of a patient in whom colonoscopy has failed to detect the source of
bleeding. This is of particular importance in an elderly patient who has evidence
of significant ongoing bleeding, but who is cardiovascularly stable. Angiography
relies on active bleeding for diagnosis, following which therapeutic embolisation
of the offending vessel may be performed. Should the investigation fail to
demonstrate the cause, and the patient continues to bleed, then laparotomy and
colectomy may be necessary as a life-saving procedure. If the bleeding is clearly
of an upper GI source (i.e. dark, altered blood or even melaena), upper GI
endoscopy would be the initial investigation of choice.

YOUR ANSWER WAS INCORRECT

Scenario 3

An anxious 20-year-old man, who has a strong family history of colorectal


cancer, presents with a positive faecal occult blood test.

C - Colonoscopy CORRECT ANSWER

Colonoscopy is a more accurate screening test than double contrast barium


enema. However, the latter is cheaper and associated with lower morbidity and
mortality rates. In view of the family history and positive FOB test, colorectal
polyps should be excluded. Polyps smaller than 1cm in size can be easily
missed by barium enema.

YOUR ANSWER WAS CORRECT

16

Theme: Abdominal pain investigations Diagnostic

A Abdominal ultrasound

B Angiography (mesenteric)

C Barium enema

D Barium meal small bowel follow-through

E CT

F Erect chest X-ray

G Colonoscopy

H Serum amylase

I Supine abdominal X-ray

For each of the patients described below, select the most useful investigation
from the list of options above. Each option may be used once, more than once,
or not at all.

Scenario 1
A 70-year-old man is admitted with severe central and epigastric pain and
vomiting. On examination he has bruising on his flanks.

H - Serum amylase CORRECT ANSWER

H Serum amylase

Serum amylase estimation would be the most appropriate investigation for the
first clinical case scenario, as the most likely diagnosis is pancreatitis. A
subsequent CT may be needed if findings are equivocal. An ultrasound scan of
the abdomen is also indicated to exclude gallstones as a cause of pancreatitis.

YOUR ANSWER WAS CORRECT

Scenario 2

An 18-year-old man has a 2-year history of central abdominal pain, which now
has moved to the right iliac fossa. He has had two episodes of loose stools over
the past 6 months.

E - CT YOUR ANSWER

G - Colonoscopy CORRECT ANSWER

G - Colonoscopy

The history of the young man is suggestive of Crohns disease and a


colonoscopy would be the most appropriate investigation.

YOUR ANSWER WAS INCORRECT


Scenario 3

A middle-aged woman presents with epigastric pain. She has shallow breathing,
tachycardia, but is normotensive. She has rheumatoid arthritis and receives
regular gold injections and takes oral steroids and diclofenac. She has taken an
increased steroid dose over the past week because of a chest infection.

F - Erect chest X-ray CORRECT ANSWER

F Erect chest X-ray

The third patient is most likely to have a perforated peptic ulcer as evidenced by
the epigastric pain, shallow respirations and tachycardia. This would be
suspected because of oral steroid and non-steroidal intake. An erect chest X-ray
would be the most appropriate investigation.

YOUR ANSWER WAS CORRECT

Scenario 4

An elderly woman presents with a 7-day history of intermittent fresh rectal


bleeding. Examination is unremarkable, and she is haemodynamically stable.
Initial blood tests reveal a microcytic anaemia.

G - Colonoscopy CORRECT ANSWER

G Colonoscopy
The most appropriate investigation in the fourth patient would be a colonoscopy.
If the colonoscopy cannot identify the cause of the bleeding then she will require
a mesenteric angiogram and possible embolisation of the bleeding vessel.

YOUR ANSWER WAS CORRECT


17

Theme: Right iliac fossa pain

A Appendicitis

B Ectopic pregnancy

C Irritable bowel syndrome

D Ovarian torsion

E Perforated duodenal ulcer

F Ruptured ovarian cyst

G Terminal ileal Crohns disease

H Ureteric stone

For each of the following situations below, select the most likely cause of the
pain from the above list. Each item may be used once, more than once, or not at
all.

Scenario 1

A 28-year-old woman developed sudden onset of pain in the right iliac fossa.
Pain was improved by walking around the examination room in the Emergency
department.

Select one... YOUR ANSWER

H - Ureteric stone CORRECT ANSWER


H Ureteric stone

There are no features here of peritonism. A patient with ureteric colic would
typically pace around when they had the pain. A patient with peritonitis caused
by a ruptured viscus would be lying still.

YOUR ANSWER WAS INCORRECT

Scenario 2

A 28-year-old woman presents with recurrent abdominal pain localised mostly to


the right iliac fossa. She opens her bowels four times a day. The stool was noted
to be of liquid consistency. Pain was reported to be worse when eating but
relieved by bowel movement. A loss of 5 kg in weight was noted.

G - Terminal ileal Crohns disease CORRECT ANSWER

G Terminal ileal Crohns disease

The history here could be one of either Crohns disease or irritable bowel
syndrome. However, the weight loss would be more consistent with Crohns
disease. It should be noted that Crohns or any inflammatory bowel problems
pathology, including diverticular disease, can produce symptoms of functional
bowel disorders: increased stool frequency, abdominal pain relieved by
defaecation and faecal urgency.

YOUR ANSWER WAS CORRECT

18

A 22-year-old female presents to her GP with a history of vague abdominal pain


and bloating. On further questioning she reports weight loss and passing
offensive stools which are difficult to flush away. As part of her investigations a
jejunal biopsy is performed. This shows crypt hyperplasia and subtotal villous
atrophy.

What is the most likely diagnosis?

Select one answer only

Bacterial overgrowth

Coeliac disease CORRECT ANSWER

Crohns disease YOUR ANSWER

Ulcerative colitis

Tropical malabsorption

YOUR ANSWER WAS INCORRECT

The Answer

Comment on this Question

Coeliac disease is a T-cell mediated autoimmune disease affecting the small


bowel caused by gluten intolerance. It causes a variety of symptoms including
steatorrhoea (due to an inability to absorb fats as evidenced in this case by
offensive stools that are difficult to flush away), abdominal pain, bloating. It is
diagnosed by a jejunal biopsy which classically shows subtotal villous atrophy
and crypt hyperplasia. The management involves a gluten-free diet which can
relieve symptoms.

19

An 18-year-old male presents to A and E after vomiting blood following a night


out for his birthday. On further questioning he admits to consuming a large
amount of alcohol and then vomiting heavily. After vomiting around 12 times he
noticed streaks of blood in his vomit.

What is the most likely diagnosis?

Select one answer only

Aortic-enteric fistula

Gastric Cancer

Mallory-Weiss tear YOUR ANSWER

Oesophageal varices

Peptic ulcer disease

YOUR ANSWER WAS CORRECT


The Answer

Comment on this Question

A Mallory-Weiss tear is a tear in the mucosal lining of the oesophagus which


classically occurs after heavily vomiting (often following an alcohol binge). Most
cases of bleeding from this are self-limiting and usually minor.

20

An 82-year-old male is recovering in ICU 4 days after an emergency repair of a


difficult juxta-renal leaking abdominal aortic aneurysm. The nurse bleeps you
because he has become tachycardic and his blood pressure has dropped to
80/50. On assessment he is very pale and the nurse reports he has just had a
large PR bleed.

What is the most likely cause for this?

Select one answer only

Angiodysplasia

Aortic-enteric fistula YOUR ANSWER

Diverticular disease

Gastric Cancer
Peptic ulcer disease

YOUR ANSWER WAS CORRECT

The Answer

Comment on this Question

Aortic-enteric fistulae occur mostly following AAA repair in which a


communication has developed between the aorta and the small bowel. They
result in very heavy blood loss into the small bowel and are often fatal.

21

Theme: Abdominal surgery

A Sigmoid colectomy
B Abdomino-perineal excision of rectum (APR)
C Hartmann's procedure
D Sub-total colectomy and formation of ileostomy
E Right hemicolectomy
F Anterior resection of rectum
G Left hemicolectomy

Choose the most appropriate procedure for the following scenarios. Each
answer may be used once, several times or not at all.

Scenario 1
A 57-year-old man in good general health presents electively with bleeding per rectum,
change in bowel habit and iron-deficiency anaemia. A caecal tumour has been
confirmed on barium enema.

E - Right hemicolectomy CORRECT ANSWER

This is the most appropriate procedure for a patient with a right-sided colonic tumour.

YOUR ANSWER WAS CORRECT

Scenario 2

A 33-year-old woman with a 12-year history of ulcerative colitis. The patient presents
as an emergency with shock. These symptoms are accompanied by severe abdominal
pain and extensive bleeding per rectum.

D - Sub-total colectomy and formation of ileostomy CORRECT ANSWER

This patient has a long history of ulcerative colitis and is presenting as an emergency
with a severe exacerbation of symptoms and a toxic megacolon. A sub-total colectomy
with formation of ileostomy should be performed after aggressive resuscitation and
administration of broad-spectrum antibiotics.

YOUR ANSWER WAS CORRECT

Scenario 3

An 85-year-old man with a history of ischaemic heart disease presents as an


emergency with severe generalised abdominal pain. Initially the pain had arisen in the
left iliac fossa and had worsened progressively over 2 days. The patient is pyrexial and
tachycardic.

G - Left hemicolectomy YOUR ANSWER

C - Hartmann's procedure CORRECT ANSWER


This patient presents with peritonitis, following a probable perforated diverticulum. A
primary anastomosis should not be performed in the presence of faecal peritonitis.
Hartmanns procedure is the most appropriate procedure in this emergency situation.

YOUR ANSWER WAS INCORRECT

Scenario 4

A 72-year-old woman who presents electively with bleeding per rectum. She
underwent a flexible sigmoidoscopy, which demonstrated a tumour in the upper
rectum.

F - Anterior resection of rectum CORRECT ANSWER

An anterior resection is appropriate for tumours of the upper rectum. Abdomino-


perineal resection is a procedure reserved for lower third tumours of the rectum.

YOUR ANSWER WAS CORRECT

22

Theme: Abdominal pain

A Leaking abdominal aortic aneurysm

B Pelviureteric obstruction

C Renal adenocarcinoma

D Ureteric colic

For each of the statements below, select the most likely diagnosis from the
above list. Each option may be used once, more than once, or not at all.

Scenario 1

A 45-year-old man presents with haematuria, loin pain and a loin mass.
C - Renal adenocarcinoma CORRECT ANSWER

C Renal adenocarcinoma

The presentation of the first patient is typically that of renal carcinoma; however,
this triad of symptoms and signs only occurs in 30% of cases.

YOUR ANSWER WAS CORRECT

Scenario 2

A 22-year-old man experiences loin pain mainly in the morning after drinking four cups
of coffee.

B - Pelviureteric obstruction CORRECT ANSWER

B Pelviureteric obstruction

Loin pain in the patient who drinks four cups of coffee before work is most likely
to be the result of pelviureteric obstruction. Symptoms of ureteric obstruction in
adults usually occur after a fluid overload.

YOUR ANSWER WAS CORRECT

Scenario 3

A 70-year-old man presents with loin pain, a pulse rate of 120/min and a BP of 80/60
mmHg.

A - Leaking abdominal aortic aneurysm CORRECT ANSWER

A Leaking abdominal aortic aneurysm

Any male patient above the age of 55 years who presents with back pain should
be suspected of having a leaking abdominal aortic aneurysm (AAA) until proven
otherwise, as AAAs are more common in this age group than urinary stones.

YOUR ANSWER WAS CORRECT

23
Theme: Haematemesis

A Acute erosive gastritis

B Aortic enteric fistula

C Dieulafoys syndrome (gastric arteriovenous malformation)

D Duodenal ulcer

E Gastric adenocarcinoma

F Gastric leiomyoma

G Gastric ulcer

H MalloryWeiss tear

I Oesophageal carcinoma

J Oesophageal varices

K Oesophagitis

L ZollingerEllison syndrome

The above are all possible causes of haematemesis. From the following clinical
scenarios please pick the most appropriate diagnosis from the list. Each item
may be used once, more than once, or not at all.

Scenario 1

A 40-year-old man presents to Casualty complaining of several episodes of coffee-


ground vomiting. He has been taking ibuprofen for 4 weeks for shoulder pain related to
an exercise-induced injury. He tells you that he has been regularly troubled by bouts of
epigastric pain over the past year. This worsens in relation to stresses at work and is
eased by eating. Full blood count: haemoglobin 11.8 g/dl, white cell count 7.4 x
109/litre, platelets 200 x 109/litre, mean corpuscular volume 84, INR 1.0.

D - Duodenal ulcer CORRECT ANSWER

D Duodenal ulcer

The history of recent non-steroidal anti-inflammatory drug (NSAID) use, on a


chronic background of dyspeptic symptoms, with the vomiting of altered blood,
points to either peptic ulcer disease or erosive gastritis. However, epigastric pain
that is relieved by eating is said to be commonly experienced by those suffering
from duodenal ulcer. Other features that are more predictive of duodenal ulcer
disease include pain that worsens when the patient is experiencing stress and
anxiety, and the presence of nocturnal dyspepsia. Investigation should be by
gastroscopy with or without endoscopic therapy if actively bleeding (emergency
not required in this case). Management involves cessation of NSAIDs and
continued medical therapy with histamine antagonists or proton pump inhibitors
with or without Helicobacter pylori eradication if necessary.

YOUR ANSWER WAS CORRECT

Scenario 2

A 67-year-old man is brought into The Emergency Department by his family. He is


vomiting small amounts of fresh blood. His wife tells you that he has been complaining
of a gnawing upper abdominal pain for nearly 2 months but has put it down to
indigestion and stress at work. She is also worried about his weight loss and poor
appetite (can only eat a little at a time). Full blood count: haemoglobin 9.4 g/dl, white
cell count 6.0 x 109/litre, platelets 390 x 109/litre, mean corpuscular volume 68, INR 1.0.

E - Gastric adenocarcinoma CORRECT ANSWER


E Gastric adenocarcinoma

The incidence of gastric cancer reaches its peak between the ages of 50 and 70
years. Men are affected two to three times more than women. Epigastric pain
with no specific relation to eating (and not relieved by antacids), early
satiety/anorexia and dramatic weight loss are all symptoms associated with the
disease. Depending on the site of the tumour other symptoms may include
dysphagia (carcinoma at the cardia leading to oesophagogastric obstruction) or
regurgitation of undigested food (cancer at the pylorus mimicking pyloric
stenosis). A low haemoglobin and mean corpuscular volume attest to iron
deficiency anaemia secondary to chronic blood loss.

YOUR ANSWER WAS CORRECT

Scenario 3

An unkempt man is brought into majors by ambulance. He is vomiting copious


amounts of fresh red blood. On examination, he is drowsy and you notice (after
initial resuscitation is instituted) that he has clinical ascites. Full blood count:
haemoglobin 7.4 g/dl, white cell count 12.0 x 10 9/litre, platelets 190 x 109/litre,
mean corpuscular volume 102, INR 1.9.

J - Oesophageal varices CORRECT ANSWER

J Oesophageal varices

Oesophageal varices result from portal venous hypertension. This is most


commonly the result of cirrhosis of the liver, usually associated with chronic
alcohol abuse (note raised mean corpuscular volume because of the impact of
alcohol on the bone marrow, and raised INR secondary to hepatic dysfunction, in
this scenario). Porto-systemic shunting occurs as resistance to flow occurs and
pressure rises in the portal venous system. In the significant, acute bleed clearly
resuscitation must be implemented first large-bore cannulae (central venous
pressure line if possible), crystalloid or colloid immediately, with blood
transfusion as soon as this is available. Fresh frozen plasma, vitamin K and
platelets can be given if a coagulopathy is established, or if a large volume of
blood needs to be replaced. Vasopressin/Terlipressin intravenously can help to
markedly reduce portal venous pressure and slow or halt bleeding to allow a
skilled endoscopist to band or inject the varices. If the bleeding remains
torrential then tamponade using a SengstakenBlakemore tube can be attempted
while definitive therapy or transfer are considered. Mortality from varices is high,
not only because of the bleeding but as a result of the other disastrous
complications of liver failure in alcoholics.

YOUR ANSWER WAS CORRECT

24

Theme: Vomiting

A Drug-related

B Central neuronal causes

C Cholecystitis

D Enteritis
E Gastritis

F Metabolic/endocrine

G Obstruction

H Pancreatitis

I Peptic ulcer disease

The following patients have all presented with nausea and vomiting. Please
select the most appropriate diagnosis from the above list. The items may be
used once, more than once, or not at all.

Scenario 1

A 23-year-old man attends The Emergency Department on a Sunday night with


nausea, vomiting and epigastric pain. On examination, there is little to find other than
mild epigastric tenderness. Investigations are normal.

E - Gastritis CORRECT ANSWER

E Gastritis

This, and enteritis, are probably the commonest causes of vomiting seen in a
typical Emergency Department. Enteritis tends to have little in the way of
epigastric pain and more central colic with or without diarrhoea.

YOUR ANSWER WAS CORRECT

Scenario 2

A 56-year-old woman is 1-day post uncomplicated laparoscopic cholecystectomy. She


is due to go home but has persistent nausea and vomiting.

F - Metabolic/endocrine YOUR ANSWER


A - Drug-related CORRECT ANSWER

A Drug-related

This lady has post-operative nausea and vomiting (PONV). The problem is
multifactorial but is principally related to anaesthetic drugs. It is three times
more common in women and is best prevented rather than treated.

YOUR ANSWER WAS INCORRECT

The Answer
It is not impossible to list approximately 150 causes of nausea and vomiting. Broadly,
however, a complete overview of all the causes of vomiting includes five groups:

central (intracranial, labyrinthine etc)

metabolic and endocrine (uraemia, pregnancy, diabetes etc)

iatrogenic (cancer chemotherapy, digoxin, opiates etc)

obstructive (any level), and

mucosal inflammation (appendicitis, gastritis, cholecystitis etc).

25

Theme: Complications of splenectomy

A Basal atelectasis

B Deep vein thrombosis

C Gastric perforation
D Gastric stasis

E Lower lobe pneumonia

F Overwhelming post-splenectomy sepsis

G Pancreatic fistula

H Pancreatic pseudocyst

I Portal vein thrombosis

J Pulmonary embolus

K Splenosis

L Subphrenic abscess

The following are descriptions of patients with post-splenectomy complications.


Please select the most appropriate diagnosis from the list. The items may be
used once, more than once, or not at all.

Scenario 1

A 13-year-old boy presents to The Emergency Department with a severe


headache and vomiting; for the past 2 to 3 days he has been off school for
presumed influenza. On examination he is flushed and has a temperature of
39C. There are no specific findings; however, on further questioning his parents
state that he had an emergency splenectomy following a road traffic accident 1
year ago.

E - Lower lobe pneumonia YOUR ANSWER

F - Overwhelming post-splenectomy sepsis CORRECT ANSWER

F Overwhelming post-splenectomy sepsis


After splenectomy the ability of the spleen to destroy encapsulated organisms is
lost and infection can lead to the distinct clinical syndrome of overwhelming
post-splenectomy infection (OPSI). In children splenectomised for trauma the
incidence is about 12%, while for adults it is about 0.5%. Long-term penicillins
are of proven value in children, as the maximal incidence of OPSI occurs within 2
years of surgery. Prophylaxis with polyvalent vaccines is also recommended
(usually to pneumococcus, meningococcus and Haemophilus influenzae).

YOUR ANSWER WAS INCORRECT

Scenario 2

A 42-year-old woman complains of left-sided chest and abdominal pain, worse on


inspiration, 7 days post-splenectomy. Examination reveals a temperature of 38.6C,
decreased air entry at the left lung base, dullness to percussion and left upper
quadrant tenderness.

A - Basal atelectasis YOUR ANSWER

L - Subphrenic abscess CORRECT ANSWER

L Subphrenic abscess

Post-operative haematomas are common post-splenectomy. They may collect in


the subphrenic space, and infection can readily lead to a subphrenic abscess.
They may cause an associated pleural effusion as in this case. Characteristically
they present with swinging pyrexia, sweats, rigors and left-sided pain on deep
inspiration. Treatment is usually by percutaneous drainage.

YOUR ANSWER WAS INCORRECT

The Answer

Comment on this Question


Some operations have a good list of specific complications that are beloved of
examiners. Favoured are probably thyroidectomy, splenectomy, laparoscopic
cholecystectomy and inguinal hernia repair.

26

A 17-year-old male is being investigated for tremor and dysarthria. His GP has
performed blood tests which show very deranged LFTs. On examination, he is
noted to have a tremor, impaired memory and slit-lamp examination reveals
Kayser-Fleischer rings.

What is the most likely diagnosis?

Select one answer only

Alcoholic cirrhosis

Alpha-1 antitrypsin deficiency

Autoimmune hepatitis

Hepatitis B

Wilsons disease YOUR ANSWER


YOUR ANSWER WAS CORRECT

The Answer

Comment on this Question

Wilsons disease is an autosomal recessive disorder that results in the toxic


accumulation of copper in the liver and CNS (in particular the basal ganglia).
Children often present with liver disease, and adults with CNS features. Kayser-
Fleischer rings are copper deposits in the iris (Descemets membrane) that are
pathognomic but not invariable and may require a slit-lamp to be seen.

27

Theme: Hernias

A Littrs hernia

B Maydls hernia

C Pantaloon hernia

D Richters hernia

E Sliding hernia

For each description listed below, select the most appropriate hernia from the
above list. Each option may be used once, more than once, or not at all.

Scenario 1

Dual sacs straddling the inferior epigastric vessels

C - Pantaloon hernia CORRECT ANSWER

C Pantaloon hernia

YOUR ANSWER WAS CORRECT


Scenario 2

Two separate loops of bowel

Select one... YOUR ANSWER

B - Maydls hernia CORRECT ANSWER

B Maydls hernia

YOUR ANSWER WAS INCORRECT

Scenario 3

The posterior wall of the hernial sac is formed by a herniating viscus

Select one... YOUR ANSWER

E - Sliding hernia CORRECT ANSWER

E Sliding hernia

YOUR ANSWER WAS INCORRECT

Scenario 4

Portion of circumference of the bowel

D - Richters hernia CORRECT ANSWER

D Richters hernia

YOUR ANSWER WAS CORRECT

Scenario 5

Meckels diverticulum

A - Littrs hernia CORRECT ANSWER

A Littrs hernia

YOUR ANSWER WAS CORRECT


28

Theme: Hernias

A Diaphragmatic hernia

B Epigastric hernia

C Gluteal hernia

D Lumbar hernia

E Obturator hernia

F Perineal hernia

G Sciatic hernia

H Spigelian hernia

For each site of herniation below, select the most likely hernial type from the
above list. Each answer may be used once, more than once, or not at all.

Scenario 1

Triangle of Petit

D - Lumbar hernia CORRECT ANSWER

YOUR ANSWER WAS CORRECT

Scenario 2

Greater sciatic notch


G - Sciatic hernia YOUR ANSWER

C - Gluteal hernia CORRECT ANSWER

YOUR ANSWER WAS INCORRECT

Scenario 3

Pelvic floor

D - Lumbar hernia YOUR ANSWER

F - Perineal hernia CORRECT ANSWER

YOUR ANSWER WAS INCORRECT

Scenario 4

Linea semilunaris

D - Lumbar hernia YOUR ANSWER

H - Spigelian hernia CORRECT ANSWER

YOUR ANSWER WAS INCORRECT

The Answer

Comment on this Question


Lumbar hernias occur through either the inferior lumbar triangle of
Petit(bounded by the iliac crest, the posterior edge of the external oblique
andthe anterior edge of the latissimus dorsi) or the superior lumbar
space(bounded by the 12th rib, the lower border of the serratus posterior
inferioras well as the anterior border of the sacrospinalis and the internal
oblique).A perineal hernia is seen as a rare complication of abdominoperineal (A-
P) resection, and develops through a non-healing perineal wound.Gluteal
hernias may occur through the greater sciatic notch, above or below the
piriformis. Spigelian hernias account for less than 1% of abdominal hernias, and
occur in the linea semilunaris on the lateral border of the rectus abdominis at the
level of the arcuate line.

29

Theme: Loin pain

A Aortic aneurysm

B Pancreatitis

C Pelviureteric junction (PUJ) obstruction

D Pyelonephritis

E Renal cell carcinoma

F Urinary bladder obstruction

G Urinary calculi

For each of the patients described below, select the most likely diagnosis from
the list of options above. Each option may be used once, more than once, or not
at all.

Scenario 1

A 30-year-old man presents with loin pain, pyrexia and tachycardia.


D - Pyelonephritis CORRECT ANSWER

D Pyelonephritis

One would suspect pyelonephritis in a male patient with loin pain, pyrexia and
tachycardia.

YOUR ANSWER WAS CORRECT

Scenario 2

An 18-year-old man presents with pain in his right iliac fossa, so intense that he finds it
impossible to remain still, and microscopic haematuria.

G - Urinary calculi CORRECT ANSWER

G Urinary calculi

In an 18-year-old man with right iliac fossa pain and microscopic haematuria,
appendicitis or a urinary calculus should be suspected. With a perforated
appendicitis, however, the patient lies still, unlike the writhing around with the
pain of ureteric colic.

YOUR ANSWER WAS CORRECT

Scenario 3

A woman known to have a previous history of bilateral reflux presents with dysuria,
fever and feeling generally unwell.

F - Urinary bladder obstruction YOUR ANSWER

D - Pyelonephritis CORRECT ANSWER

D Pyelonephritis
Here the most likely diagnosis is pyelonephritis in view of the bilateral reflux,
dysuria, malaise and fever.

YOUR ANSWER WAS INCORRECT

30

Theme: Splenomegaly

A Brucellosis
B Infectious mononucleosis
C Sickle cell disease
D Lymphoma
E Leukaemia
F Spherocytosis
G Polycythaemia rubra vera

Which of the above diagnoses fit the following clinical scenarios? Select the
single most likely condition from the options listed above. Each option may be
used once, more than once or not at all.

Scenario 1

A young man with splenomegaly fever and titres positive to EpsteinBarr virus (EBV)

B - Infectious mononucleosis CORRECT ANSWER

Infectious mononucleosis or glandular fever is a common disease of young adults and


may present with a sore throat, lethargy, fever, lymphadenopathy, splenomegaly and
hepatitis. It is caused by EBV.
YOUR ANSWER WAS CORRECT

Scenario 2

An 18-year-old White girl with gallstones, splenomegaly and jaundice

F - Spherocytosis CORRECT ANSWER

Spherocytosis is a cause of haemolytic anaemia and can lead to jaundice, gallstones


and splenomegaly.

YOUR ANSWER WAS CORRECT

Scenario 3

An Afro-Caribbean nurse with joint pains, splenomegaly and low haemoglobin (HB)

C - Sickle cell disease CORRECT ANSWER

Sickle cell disease can cause severe haemolysis. There is typically anaemia, jaundice
and splenomegaly. Crisis can lead to severe bone pain.

YOUR ANSWER WAS CORRECT

31

THEME: Abdominal pathologies

A. Crohns disease

B Mid-menstrual pain (mittelschmerz)

C Endometriosis
D Carcinoma of the rectum

E Ulcerative colitis

F Torsion of the ovary

G Diverticular disease

H Ruptured ectopic pregnancy

I Mesenteric infarction

J Carcinoma of the ileocaecal region

For each of the case descriptions below, select the most appropriate diagnosis
from the list above. Each option may be used once, more than once, or not at all.

Scenario 1

A 73-year-old woman presents to the Accident and Emergency Department with


lower abdominal distension and pain relieved by passing flatus. She reports a
history of altered bowel habits and a sense of incomplete evacuation of her
bowels; her stools are mixed with blood.

D - Carcinoma of the rectum CORRECT ANSWER

Colonic malignancy is the second most common cause of cancer deaths in the UK.
Predisposing factors include

1/neoplastic polyps,2/ ulcerative colitis, 3/familial polyposis and a 4/positive family


history.

Clinical presentation depends on the site: left-sided colonic carcinoma presents with
abdominal pain (relieved by passing flatus), abdominal distension, per-rectal bleeding,
altered bowel habits and tenesmus; a mass may be felt on per-rectal examination. In
right-sided tumours, the patient may present with symptoms of anaemia from occult
bleeding. Sometimes the patient complains of pain in the right iliac fossa and per-
abdominal examination may reveal a mass over this region.

YOUR ANSWER WAS CORRECT

Scenario 2

A 30-year-old woman presents to the surgical outpatient clinic with a 4-month history of
diarrhoea and abdominal colic. She has recently lost weight (half-a-stone) and has
noticed some ulcers in her mouth. She is a smoker. Her blood test reveals microcytic,
hypochromic anaemia. Colonoscopy demonstrates skip lesions with a cobblestone
mucosal appearance.

A - Crohns disease CORRECT ANSWER

Crohns disease is common in North America and Northern Europe . Unlike ulcerative
colitis, Crohns disease affects the whole of the gastrointestinal tract. Risk factors
include a strong positive family history, various types of food, smoking (increases the
risk by three fold), and infective agents such as mycobacterium and cell-wall-deficient
organisms such as pseudomonas. Acute Crohns disease may simulate acute
appendicitis but is usually preceded by diarrhoea. In chronic Crohns disease, mild
diarrhoea is experienced over many months, accompanied by intestinal colic;
intermittent fevers, secondary anaemia and weight loss are also common. With
progression of the disease, adhesions, transmural fissures, intra-abdominal abscesses
and fistulous tracts may develop.

It is important to note that endoscopic findings in Crohns disease include skip lesions
and a cobblestone appearance of the gut mucosa (with occasional stricturing), all of
which are less common in ulcerative colitis. Candidate must be aware of all endoscopic
and histological features of both Crohns disease and ulcerative colitis.
YOUR ANSWER WAS CORRECT

Scenario 3

A 21-year-old woman is brought to the Accident and Emergency Department with


severe generalised lower abdominal pain. She is pale and tachycardic and her blood
pressure is 90/54 mm Hg.

H - Ruptured ectopic pregnancy CORRECT ANSWER

Ectopic pregnancy occurs in less than 1% of pregnancies. The typical history of ectopic
pregnancy is one or two missed menstrual periods with other signs of pregnancy
(mastalgia, morning sickness and increased urinary frequency). In ruptured ectopic
pregnancy, the abdominal pain is initially crampy, but subsequently becomes a more
continuous and generalised lower abdominal pain. Irritation of the diaphragm may lead
to shoulder-tip pain. There may be signs of hypotension and hypovolaemic shock. Per-
abdominal examination may reveal guarding, rigidity and rebound tenderness.
Frequently, altered blood may be seen in the cervix and movement of the cervix
produces abdominal discomfort. In some instances, a mass may be felt in one of the
adenexae and the uterus is frequently soft and bulky. Ruptured ectopic pregnancy
warrants immediate surgical intervention.

YOUR ANSWER WAS CORRECT

Scenario 4

An 83-year-old woman presents with a 12-hour history of severe generalised


abdominal pain associated with nausea and vomiting. Her blood pressure is 100/70
mm Hg. She is in atrial fibrillation. Bowel sounds are absent.

I - Mesenteric infarction CORRECT ANSWER


Although any of the three anterior abdominal aortic branches (coeliac, superior and
inferior mesenteric vessels) may occlude, it is the occlusion of the superior mesenteric
artery (SMA) that commonly causes mesenteric infarction. Despite the presence of
collateral vessels of the SMA, these may not be able to dilate sufficiently and swiftly to
overcome the acute reduction in blood flow. The occlusion may be due to a thrombus
or an embolus and is seen in elderly patients who are in atrial fibrillation. Other less
common causes of infarction include dissecting aneurysm and vasculitis. Clinical
features include persistent, severe and generalised abdominal pain. The inflammatory
markers may be elevated and the blood gas may reveal a metabolic acidosis. This
condition is a surgical emergency, as the patient rapidly becomes toxic and may die
from septic shock unless the infarcted bowel (dead gut) is removed.

YOUR ANSWER WAS CORRECT

32

A previously well 83-year-old woman is brought into A and E with abdominal pain
and vomiting. On examination, she has a distended abdomen, and denies any
previous surgery. On assessment of her groins, a tender lump is palpable in the
left groin. It is below and lateral to the pubic tubercle.

What is the most likely cause of her symptoms?

Select one answer only

Epigastric hernia

Femoral hernia YOUR ANSWER

Direct inguinal hernia


Indirect inguinal hernia

Obturator hernia

YOUR ANSWER WAS CORRECT

The Answer
Comment on this Question

Femoral hernias classically emerge below and lateral to the pubic tubercle, and are
more common in women due to the wider pelvis. Inguinal hernias emerge from above
and medial to the pubic tubercle and come in two forms - direct (which pass through
Hasselbachs triangle) and indirect (which enter via the deep inguinal ring). As the neck
for femoral hernias is relatively narrow and stiff, they are more likely to obstruct and
strangulate.

33

A 27-year-old male who works in the city presents with crampy abdominal pain
and a 6 week history of bloody diarrhoea with mucus. He is referred for
investigation.Sand a sigmoidoscopy reveals an inflamed mucosa and a rectal
biopsy shows an inflammatory infiltrate with goblet cell depletion and crypt
abscesses.

What is the most likely diagnosis?

Select one answer only

Coeliac disease
Crohns disease

Irritable bowel syndrome

Ulcerative colitis YOUR ANSWER

Whipples disease

YOUR ANSWER WAS CORRECT

The Answer

Comment on this Question

Ulcerative colitis commonly presents between the ages of 15 and 30 years. It is


characterised by diarrhoea often with mucus and blood. There may be a crampy
abdominal pain, and the frequency of motions may be related to the severity of
an attack. It may affect only the rectum (proctitis in approximately 50%), or
spread proximally to involve the rest of the colon. Sigmoidoscopy shows an
inflamed and friable mucosa, and histologically there may be1/ an inflammatory
infiltrate, 2/goblet cell depletion,3/ glandular distortion, 4/mucosal ulcers and
5/crypt abscesses.

34

A 62-year-old male presents to his GP as his family have noticed he has gone
yellow. On further questioning he admits to generalised pruritus and around 2
stone of weight loss. He denies any abdominal or chest pain. On examination
you note he has icteric sclera and widespread scratch marks.

What is the most likely diagnosis?

Select one answer only

Biliary colic

Cholangitis

Cholecystitis

Gallstone ileus

Pancreatic Cancer YOUR ANSWER

YOUR ANSWER WAS CORRECT

The Answer

Comment on this Question

This patient has several features of jaundice- icteric sclera and generalised
pruritus, other questions which would be useful in the history would be
regarding any colour change in his urine or stools. Obstructive jaundice
classically produces dark urine and paler stools. The absence of pain goes
against the first four options and the significant recent weight loss suggests a
malignant process may be present. In any patient with a history of painless,
obstructive jaundice, the diagnosis of Pancreatic Ca must be considered.

35

An 18-year-old A-level student is referred from his GP with RIF pain and nausea.
On examination, she has guarding and tenderness in the RIF. On deep palpation
in the LIF she complains of pain in the RIF.

What is this sign called?

Select one answer only

Aarons sign

Dunphys sign

Murphys sign

Riglers sign

Rosvings sign YOUR ANSWER

YOUR ANSWER WAS CORRECT

The Answer
Rosvings sign describes pain in the RIF more than the LIF when the LIF is
pressed, and is pathognomic of appendicitis. Aarons sign is pain referred to the
epigstrium upon continuous firm pressure over McBurneys point and is
indicative of appendicitis. Dunphys sign is a medical sign in which increased
abdominal pain occurs with coughing and is often present in appendicitis.
Murphys sign occurs when pressing over the RUQ with two fingers and asking
the patient to breathe in, causes pain and arrest of inspiration as the inflamed
gall bladder impinges of the examiners fingers. It is only positive if repeating the
test on the left does not result in a similar pain. Riglers sign is a radiologic sign
in which air is seen on an abdominal X ray on both sides of the intestine.

36

Theme: Acute abdominal pain


A Acute appendicitis

B Acute cholecystitis

C Acute pancreatitis

D Biliary colic

E Intestinal obstruction

F Obstructive jaundice

G Perforated peptic ulcer

H Ruptured abdominal aortic aneurysm

I Ruptured liver

J Ruptured spleen
For each of the following situations, select the most appropriate cause of acute
abdominal pain from the above list. Each option may be used once, more than
once, or not at all.
Scenario 1
A 43-year-old barmaid complains of severe epigastric/right upper quadrant (RUQ) pain
2 hours after eating a meal of chips and fried chicken. The pain radiates to her back
and makes her nauseated. There are no other systemic symptoms.

Select one... YOUR ANSWER


D - Biliary colic CORRECT ANSWER
D Biliary colic
Biliary colic presents with pain in the RUQ/epigastric region usually (but not
invariably) 23 hours after a fatty meal. The pain is sometimes associated with
nausea and vomiting. There are no inflammatory signs, such as raised
temperature or increased WCC, which may be present when the obstruction
leads to acute cholecystitis.

YOUR ANSWER WAS INCORRECT

Scenario 2
A 38-year-old executive of Asian origin is brought to the Emergency Department after
his car was involved in a high-speed RTA. He presents with upper abdominal
distension, guarding and signs of shock. He also complains of left shoulder tip pain and
his past medical history includes recurrent episodes of malaria.

J - Ruptured spleen CORRECT ANSWER


J Ruptured spleen
Rupture of the spleen is usually the result of a closed injury caused by direct
external violence, typically by the steering wheel of a car or the handlebars of a
bicycle. At all ages, splenic enlargement from malaria, infectious
mononucleosis, vascular malformations and haematological malignancies all
predispose the spleen to rupture following trivial trauma. Clinical features
include signs of internal haemorrhage and pain referred to the left shoulder
because of irritation of the diaphragm (Kehrs sign). Local signs include upper
abdominal guarding, local bruising and abdominal distension.

YOUR ANSWER WAS CORRECT

Scenario 3
A 78-year-old lady presents with severe abdominal pain and signs of peritonism. She is
on non-steroidal anti-inflammatory agents (NSAIDs) for rheumatoid arthritis. On
examination, bowel sounds are absent. Erect chest X-ray shows free gas under the
diaphragm.

G - Perforated peptic ulcer CORRECT ANSWER


G Perforated peptic ulcer
Perforated peptic ulcer is common in patients who are on regular longterm
NSAIDs. Perforation of a viscus causes sudden, severe pain which is usually
appreciated in the area of the perforation upper epigastric in the case of
duodenal ulcer. There may be shoulder tip pain if the diaphragmatic peritoneum
is inflamed. The patient may lie motionless and supine with shallow respiratory
excursions. An erect chest X-ray reveals free gas under the diaphragm. Serum
amylase may be mildly elevated (> 100 iu/ml) in some patients. Surgical
intervention is required and the patient must be fully resuscitated before surgery
with intravenous fluids, nasogastric tube, adequate analgesia and appropriate
antibiotics. Surgery involves patching the perforation with greater omentum or
performing a direct closure.

YOUR ANSWER WAS CORRECT

Scenario 4
A 74-year-old man is brought to the Emergency Department with sudden onset of
upper abdominal and periumbilical pain radiating to his back. He is sweating,
nauseated and feeling faint. His BP is 90/60 mmHg and his pulse rate is 110/min.
H - Ruptured abdominal aortic aneurysm CORRECT ANSWER
H Ruptured abdominal aortic aneurysm
Risk factors for aortic aneurysm include: male sex, advancing age, hypertension,
tobacco smoking, chronic obstructive airways disease (irrespective of tobacco
smoking), infection of the vessel (eg salmonella), and occlusive arterial disease
affecting the coronary, carotid and the limb arteries. A diagnosis of leaking or
ruptured abdominal aortic aneurysm should be considered in the elderly with
sudden onset of upper abdominal/periumbilical pain radiating to the back and
associated with signs of progressive shock. Collapse is common and the patient
is often pale and clammy with a low volume pulse. Acute pancreatitis is the other
diagnosis to be considered however given the age and sex of the patient aortic
aneurysm is more likely here. In ruptured aortic aneurysm, immediate surgical
intervention is mandatory.

YOUR ANSWER WAS CORRECT

37

Theme: Hernias

A Diaphragmatic hernia

B Epigastric hernia

C Gluteal hernia

D Lumbar hernia

E Obturator hernia
F Perineal hernia

G Sciatic hernia

H Spigelian hernia

For each site of herniation below, select the most likely hernial type from the
above list. Each answer may be used once, more than once, or not at all.

Scenario 1

Triangle of Petit

D - Lumbar hernia CORRECT ANSWER

D Lumbar hernia

YOUR ANSWER WAS CORRECT

Scenario 2

Greater sciatic notch

C - Gluteal hernia CORRECT ANSWER

C Gluteal hernia

YOUR ANSWER WAS CORRECT

Scenario 3

Pelvic floor

F - Perineal hernia CORRECT ANSWER

F Perineal hernia
YOUR ANSWER WAS CORRECT

Scenario 4

Linea semilunaris

H - Spigelian hernia CORRECT ANSWER

H Spigelian hernia

YOUR ANSWER WAS CORRECT

The Answer

Comment on this Question


Lumbar hernias occur through either the inferior lumbar triangle of Petit
(bounded by the iliac crest, the posterior edge of the external oblique and the
anterior edge of the latissimus dorsi) or the superior lumbar space (bounded by
the 12th rib, the lower border of the serratus posterior inferior as well as the
anterior border of the sacrospinalis and the internal oblique). A perineal hernia is
seen as a rare complication of abdominoperineal (A-P) resection, and develops
through a non-healing perineal wound.

38

Theme: Colorectal operations

A Transverse loop colostomy

B Hartmanns procedure

C Sigmoid colectomy and primary anastomosis

D Subtotal colectomy with ileostomy


E Panproctocolectomy

F Anterior resection

Select the most appropriate operation for the patients below. Each option may
be used once, more than once or not at all.

Scenario 1

An 85-year-old man presented with large bowel obstruction. Following resuscitation he


under went a laparotomy where a large carcinoma of the rectum was found fixed to the
lateral pelvic wall with omental and peritoneal secondary deposits.

A - Transverse loop colostomy CORRECT ANSWER

A sigmoid colectomy and primary anastomosis would not address the distal
obstruction. A subtotal colectomy, panproctocolectomy and anterior resection
would not be possible because of pelvic fixity and also would be of not curative
value as the tumour is metastatic.

YOUR ANSWER WAS CORRECT

Scenario 2

A 75-year-old man presented with 18 hours of acute abdominal pain. He was noted to
have generalised peritonitis and free intra-peritoneal air on erect chest X-ray (CXR). At
laparotomy he was found to have a widespread faecal peritonitis due to perforated
diverticular sigmoid disease.

B - Hartmanns procedure CORRECT ANSWER

Primary anastomosis can be performed at the time of emergency surgery if


colonic lavage is carried out. However, in cases of widespread faecal peritonitis,
the majority of surgeons would not embark upon such a procedure as the risk of
infection and anastomotic leak is very high. In addition, the operation time is
increased.

A Hartmanns procedure is the best operation which would aim to resect the
pathology and defunction the colon. A transverse loop would also be practical
but not ideal as the pathology would remain. The other options would not
address the problem or would be 'overkill'

YOUR ANSWER WAS CORRECT

Scenario 3

A 45-year-old man with known severe ulcerative colitis which has not responded to
maximal medical therapy. Plain abdominal X-ray (AXR) reveals a colonic diameter of 7
cm with generalised guarding. He has had 5 days of bloody diarrhoea (opening his
bowels six times per day).

D - Subtotal colectomy with ileostomy CORRECT ANSWER

The emergency surgical management of acute severe colitis in inflammatory


bowel disease includes subtotal colectomy with end ileostomy (i.e. to defunction
the inflamed distal bowel. See
http://www.bsg.org.uk/images/stories/docs/clinical/guidelines/ibd/ibd_2011.pdf)
In elective situations, panproctocolectomy would be the operation of choice.

YOUR ANSWER WAS CORRECT

39
Theme: Hernias (types and taxonomy)

A Femoral hernia

B Gluteal hernia

C Incarcerated hernia

D Incisional hernia

E Internal hernia

F Littres hernia

G Lumbar hernia

H Maydls hernia

I Obstructed hernia

J Obturator hernia

K Richters hernia

L Spigelian hernia

M Strangulated hernia

The following patients all have hernias. From the list above, select the most
appropriate diagnosis, according to the clinical and/or anatomical information
provided. The items may be used once, more than once, or not at all.

Scenario 1

A 48-year-old gentleman presents with a 36-h history of vomiting, central abdominal


pain and distension. On examination, he is unwell, febrile, dehydrated and tachycardic.
Abdominal examination reveals a distended abdomen with tinkling bowel sounds, and
a very large right inguino-scrotal hernia that is irreducible, erythematous and tender. At
operation, a W-loop of small bowel lies in the hernial sac.

M - Strangulated hernia YOUR ANSWER

H - Maydls hernia CORRECT ANSWER

H Maydls hernia

Maydls hernia is a complication of large hernial sacs, especially right


scrotal hernias in Africans. It is characterised by a W-loop of small bowel lying
in the sac, with strangulation of the intervening loop within the main abdominal
cavity by the constriction of the neck of the sac. The description of the operative
findings differentiates this hernia from afferent loop strangulation (afferent loop
entwined about afferent and efferent loops), a Richters hernia (part of the bowel
at the anti-mesenteric margin becomes strangulated), and a Littres hernia
(strangulation of a Meckels diverticulum). The operative findings also
differentiate from simple strangulation, which is associated with a similar
clinical presentation, ie evidence of gut ischaemia (severe pain with systemic
upset, eg fever, tachycardia) and obstruction (vomiting, and abdominal pain and
distension).

YOUR ANSWER WAS INCORRECT

Scenario 2

A 49-year-old woman is admitted with acute small bowel obstruction. She reports a 2-
day history of severe pain radiating down the inner aspect of the right thigh to her knee.
Consequently, you meticulously examine the hernial orifices in the groin, but no hernia
is evident. Ultimately, she requires a laparotomy, as her obstruction fails to resolve with
conservative management. Only then does the cause of her obstruction become
apparent.
B - Gluteal hernia YOUR ANSWER

J - Obturator hernia CORRECT ANSWER

J Obturator hernia

Obturator hernias are six times more common in women than men, and three
times more common after 50 years age than before. A preoperative diagnosis is
rarely made, because a swelling is not always palpable in the thigh. Therefore, it
is usually diagnosed during laparotomy for nonresolving small bowel
obstruction, as in the case described. Consequently, the operative mortality is
approximately 30%. The peritoneum protrudes through the obturator canal, and
then between the pectineus and abductor longus muscles to enter the femoral
triangle. The HowshipRomberg sign of pain referred along the geniculate
branch of the obturator nerve to the inner aspect of the knee should raise the
suspicion of an obturator hernia. Other examples of hernias frequently only
discovered during laparotomy for relief of intestinal obstruction include: gluteal
and sciatic hernias (protruding through the greater and lesser sciatic notches,
respectively), pelvic hernias (of the pouch of Douglas into the posterior wall of
the vagina or vulva; not rectocoele or cystocoele, which are false hernias), and
pudendal hernias (lateral protrusion of peritoneum through a persistent hiatus of
Schwalbe between the origin of the levator ani from the obturator internus,
usually following surgical removal of pelvic organs).

YOUR ANSWER WAS INCORRECT

Scenario 3

A 65-year-old gentleman attends the outpatients department with a long history of an


intermittent swelling in the right groin that is now persistent. The swelling is irreducible
but is non-tender, and lies above and medial to the pubic tubercle. He denies any
gastrointestinal disturbance.
K - Richters hernia YOUR ANSWER

C - Incarcerated hernia CORRECT ANSWER

C Incarcerated hernia

This question draws the candidates attention to the clinical differentiation of


complications of hernias. Incarceration refers to fixation of contents within the
hernia sac as a result of adhesions. Such a hernia is irreducible, but is neither
tender, nor associated with systemic upset (differentiating it from a strangulated
hernia), nor associated with gastrointestinal symptoms (differentiating it from an
obstructed hernia).

YOUR ANSWER WAS INCORRECT

40

Theme: Conditions that may require splenectomy

A Autoimmune haemolytic anaemia

B Chronic myeloid leukaemia

C Congenital spherocytosis

D Feltys syndrome

E Gauchers disease

F Hodgkins lymphoma

G Idiopathic thrombocytic purpura

H Myelofibrosis
I Splenic abscess

J Thalassaemia

K Trauma

L Tropical splenomegaly

The following are descriptions of patients with splenomegaly. Please select the
most appropriate diagnosis from the list. The items may be used once, more
than once, or not at all.

Scenario 1

A 13-year-old boy is referred with intermittent colicky right upper quadrant pain.
Examination reveals clinical anaemia, jaundice and splenomegaly. Abdominal
ultrasound reveals a moderately enlarged spleen and multiple, small gallstone calculi.

C - Congenital spherocytosis CORRECT ANSWER

C Congenital spherocytosis

Approximately 75% of cases are autosomal dominant disorders characterised by


defects in the red cell membrane that render erythrocytes spheroidal, less
deformable and vulnerable to splenic sequestration. Anaemia, moderate
splenomegaly and jaundice are characteristic. After 10 years of age, 4050% of
patients develop gallstones secondary to hyperbilirubinaemia. Splenectomy is
indicated in the majority of patients, as red cell survival reverts to normal even
though the abnormality persists.

YOUR ANSWER WAS CORRECT

Scenario 2
A 50-year-old man, with a 3-year history of fatigue, weight loss and anorexia, is
referred for recent onset of a dragging sensation in the upper abdomen. Examination
reveals generalised lymphadenopathy and massive splenomegaly.

B - Chronic myeloid leukaemia CORRECT ANSWER

B Chronic myeloid leukaemia

This primarily affects adults in the 4th and 5th decades of life, accounting for 15
20% of all cases of leukaemia. It is a clonal disorder of pluripotent stem cells that
predominantly differentiate along the granulocytic pathway. Initial symptoms are
non-specific; however, a dragging sensation in the abdomen caused by extreme
splenomegaly is characteristic. Symptomatic hypersplenism may occur during
the chronic phase of the disorder and may require splenectomy.

YOUR ANSWER WAS CORRECT

Scenario 3

A 26-year-old woman, with a history of intravenous drug abuse, presents with severe
upper left abdominal pain, fever and rigors. On examination she is tachycardic and
pyrexial with a mildly enlarged tender spleen.

I - Splenic Abscess CORRECT ANSWER

I - Splenic Abscess

Splenic abscesses are rare, probably because of the spleens exceptional ability
to cope with septic and foreign material. They may, however occur as a result of
haematogenous spread as in this case with the clinical triad of left upper
quadrant pain, fever and systemic signs of sepsis. Treatment is usually by
percutaneous drainage of the abscess and treatment of the cause.

YOUR ANSWER WAS CORRECT


The Answer
There are a myriad of causes of splenomegaly which the diagnostic sieve
approach can usually resolve. The list above includes some of the more
common faced by the surgeon for consideration of splenectomy. The main
indication outside trauma (blunt or iatrogenic) for splenectomy is
hypersplenism, which is a rather indefinite syndrome characterised by
enlargement, and any combination of anaemia, leukopenia and
thrombocytopaenia, bone marrow hyperplasia and improvement after
splenectomy.

41

Theme: Chronic non-malignant pelvic pain in women

A Benign ovarian tumour

B Complications of uterine leiomyomata

C Chronic constipation

D Degenerative changes of the lumbar spine

E Diverticular disease

F Endometriosis

G Inflammatory bowel disease

H Irritable bowel syndrome

I Painful bladder syndrome

J Pelvic adhesions
K Pelvic congestion syndrome

L Pelvic inflammatory disease

M Recurrent cystitis

N Urolithiasis

O Utero-vaginal prolapse

The following female patients present with chronic non-malignant pelvic pain.
From the list above, select the most likely diagnosis. The items may be used
once, more than once, or not at all.

Scenario 1

A 58-year-old librarian attends outpatients with a 12-month history of pelvic pain. She
describes the pain as dragging in nature, and reports that it is worse towards the end
of her working day. She also complains of difficulty voiding, and a sensation of
incomplete emptying following defaecation. She had four children before undergoing a
premature menopause at the age of 40 years. She takes no regular medication.

O - Utero-vaginal prolapse CORRECT ANSWER

O Utero-vaginal prolapse

Utero-vaginal prolapse is related to pelvic floor trauma sustained during


childbirth and oestrogen deficiency following the menopause. Typically,
symptoms associated with genital prolapse include a dragging sensation in the
pelvis or awareness of something coming down. Resultant pelvic floor
dysfunction impacts on urinary, sexual and rectal evacuatory function, resulting
in voiding difficulties/urinary incontinence, impaired coitus and evacuation
difficulties, respectively. The multi-system disruption of pelvic organ function
suggests the diagnosis, which is confirmed on clinical examination. Such
patients are prone to bladder and rectal prolapse as a result of weakening of the
pelvic floor musculature. Treatment may include non-surgical measures (pelvic
floor physiotherapy, oestrogen replacement therapy and intravaginal devices),
although surgery to eliminate the prolapse and restore pelvic floor function
offers definitive treatment.

YOUR ANSWER WAS CORRECT

Scenario 2

A 24-year-old woman presents with a chronic history of pelvic pain following the birth of
her first child. She has no relevant lower gastrointestinal, urological, or gynaecological
symptoms. Abdominal examination of the abdomen reveals a non-tender mass in the
left iliac fossa. There is no clinical evidence of ascites or organomegaly.

A - Benign ovarian tumour CORRECT ANSWER

A Benign ovarian tumour

Benign ovarian tumours are common in this age group. Most tumours of the
ovary are simply large versions of the cysts that form during the normal ovarian
cycle, and so are small, asymptomatic and resolve spontaneously. These are
termed functional cysts. Benign germ cell tumours (eg dermoid cyst) are also
common in women less than 30 years of age. Other histological types include
benign epithelial and sex cord stromal tumours. Symptoms tend not to occur
until the tumour is larger than 35 cm, although complications such as rupture,
haemorrhage, or infection may result in acute pelvic pain. Occasionally, large
tumours may give rise to urinary or gastrointestinal symptoms because of
pressure effects. Menstrual disturbance is uncommon, unless the tumour
secretes oestrogens (sex cord tumours). Cyst formation may complicate
endometriosis (endometrioma), but here the clinical picture is usually of
dysmenorrhoea, dyspareunia and sub-fertility.
YOUR ANSWER WAS CORRECT

Scenario 3

A 42-year-old woman is referred by her general practitioner (GP) with a 3-year history
of disabling supra-pubic pain that has resulted in repeated absence from work. Her GP
states that her urinalysis and repeated urine cultures have all been negative, and that
she has no relevant gynaecological history of note. The GP suspects that her pain is
the result of bowel pathology, and asks for your assessment. On direct questioning,
the patient describes urinary frequency and urgency. She has no gastrointestinal
symptoms. Examination is unremarkable.

M - Recurrent cystitis YOUR ANSWER

I - Painful bladder syndrome CORRECT ANSWER

I Painful bladder syndrome

Patients with painful bladder syndrome complain of bladder pain and irritative

bladder symptoms (frequency, urgency, nocturia and dysuria). The


bladder pain is worse when the bladder is distended, and often improved
following micturition. Urine cultures are repeatedly negative, leading the unwary
to exclude a urinary cause of pain in such patients. Causes include radiation,
chemical and interstitial cystitis. Urine should be cultured for atypical organisms
(Mycoplasma and Ureaplasma). Cystometry and cystoscopy are useful
diagnostic tests and often reveal a low capacity, non-compliant (stiff) bladder.
Cystoscopy may reveal subepithelial haemorrhages in patients with interstitial
cystitis and allows biopsies to be taken for histological confirmation of this
condition.

YOUR ANSWER WAS INCORRECT

Scenario 4
A 65-year-old woman presents with a long history of constipation and abdominal
bloating. More recently, she has been troubled with severe intermittent left iliac fossa
pain. Looking through her notes, you notice a recent admission with brisk bleeding per
rectum. She tells you that she is awaiting a colonoscopy. She is post-menopausal and
has no urinary symptoms. Current medication includes Fybogel and hormone
replacement therapy.

B - Complications of uterine leiomyomata YOUR ANSWER

E - Diverticular disease CORRECT ANSWER

E Diverticular disease

Patients with diverticular disease commonly complain of episodic lower


abdominal pain, although some authorities question the existence of painful
diverticular disease. The pain may be associated with a change in bowel habit
and abdominal bloating, making clinical differentiation from irritable bowel
syndrome difficult. A positive history of brisk bleeding per rectum supports a
diagnosis of diverticular disease, because haemorrhage is another of the
complications of this disease. The history is not that of inflammatory bowel
disease.

YOUR ANSWER WAS INCORRECT

The Answer

Comment on this Question


There are numerous causes of chronic non-malignant pelvic pain in women. For
convenience, they may be classified anatomically according to the contents of
the pelvis, ie lower urinary tract disorders, disorders of the reproductive organs,
gastrointestinal disorders. In addition, it is important to consider that pain may
be musculo-skeletal, neurogenic, or psychological in origin. Accordingly, the list
of causes provided in the question may be classified to produce a differential
diagnosis for chronic pelvic pain that incorporates the commonest causes for
each system. The surgical trainee should be alert to causes originating
outside the gastrointestinal system, so that patients may be referred to the
appropriate specialist for further assessment.

42

Theme: Diagnosis of presenting symptoms

A Acute diverticulitis
B Ectopic pregnancy
C Mesenteric adenitis
D Meckel's diverticulum
E Intussusception
F Acute appendicitis
G Mittelschmerz

From the list above, choose the most appropriate diagnosis for the following
scenarios. Each answer may be used once, several times or not at all.

Scenario 1

A 6-month-old child with colicky abdominal pain, bilious vomiting and passage of
redcurrant jelly stools. There is a palpable mass in the right hypochondrium.
E - Intussusception CORRECT ANSWER

This child is likely to be experiencing an intussusception. This occurs most


commonly between the ages of 3 months and 1 year. The child usually presents
with colicky abdominal pain which causes marked distress, but between
episodes the child may fall asleep.

YOUR ANSWER WAS CORRECT

Scenario 2

A 14-year-old boy presents with a 12-h history of anorexia, vomiting and central
abdominal pain that has since localised to the right iliac fossa. There is guarding and
rebound tenderness.

F - Acute appendicitis CORRECT ANSWER

This history is classical of acute appendicitis. It is important to exclude a urinary


tract infection and any other source of the symptoms, including mesenteric
adenitis. A good history and examination aids in this.

YOUR ANSWER WAS CORRECT

Scenario 3

A 27-year-old female with a 1-day history of sharp right iliac fossa pain that is
continuous and severe. Her urine pregnancy test is positive. Her last menstrual period
was 8 weeks ago.

B - Ectopic pregnancy CORRECT ANSWER

The immediate suspicion in this case is of an ectopic pregnancy. Full supportive


measures should be undertaken and early consultation from a gynaecologist is
paramount.
YOUR ANSWER WAS CORRECT

Scenario 4

A 57-year-old female with a 2-day history of lower abdominal pain, maximal in the left
iliac fossa. She has a low grade pyrexia and an elevated white cell count and C-
reactive protein.

Select one... YOUR ANSWER

A - Acute diverticulitis CORRECT ANSWER

Left iliac fossa pain along with a pyrexia and elevated white cell count and C-
reactive protein are suggestive of acute diverticulitis. The patient may also
describe altered bowel habit and passage of blood per rectum.

YOUR ANSWER WAS INCORRECT

43

Theme: Abdominal pain investigations (diagnostic)

A Abdominal ultrasound

B Angiography (mesenteric)

C Barium enema

D Barium meal and follow-through

E CT

F Erect chest X-ray


G FBC

H Serum amylase

I Supine abdominal X-ray

For each of the patients described below, select the most likely investigation
from the list of options above. Each option may be used once, more than once,
or not at all.

Scenario 1

A 75-year-old female orthopaedic patient on steroids for COPD and diclofenac for pain
presents with acute, sudden-onset epigastric pain.

F - Erect chest X-ray CORRECT ANSWER

F Erect chest X-ray

The first patient is most likely to have a perforated duodenal ulcer and an erect
chest X-ray would be the best investigation.

YOUR ANSWER WAS CORRECT

Scenario 2

A 70-year-old psychiatric patient presents with periumbilical discomfort, gross


abdominal distension and absolute constipation but no vomiting.

C - Barium enema YOUR ANSWER

I - Supine abdominal X-ray CORRECT ANSWER

I Supine abdominal X-ray


The most likely diagnosis in the second patient would be a sigmoid volvulus and
the supine X-ray would show an omega loop.

YOUR ANSWER WAS INCORRECT

Scenario 3

A 34-year-old woman with severe asthma was started on steroids. She later presents
with acute-onset epigastric pain and vomiting. On examination she has decreased
bowel sounds, guarding and rigidity.

F - Erect chest X-ray CORRECT ANSWER

F Erect chest X-ray

The most likely diagnosis in the third patient is a perforated peptic ulcer and an
erect chest X-ray would be most appropriate. Air is seen under the diaphragm in
90% of cases.

YOUR ANSWER WAS CORRECT

44

Theme: Abdominal pain

A Acute appendicitis

B Ischaemic bowel

C Porphyria

D Mesenteric adenitis
E Small bowel obstruction

F Urinary tract infection

G Crohns disease

H Diverticulitis

For each of the patients below, chose the most appropriate diagnosis from the
list above. Each may be used once, more than once or not at all.

Scenario 1

A 20-year-old man presented with a 6 hour history of severe RIF ( right iliac fossa) pain
and diarrhoea. He had been admitted 12 months previously for drainage of a perianal
abscess, following which he had suffered from persistent perianal discharge.

G - Crohns disease CORRECT ANSWER

Crohns disease is a chronic inflammatory condition affecting the large and


small bowel. Patients usually present between 15 and 55 years of age. Weight
loss is common. Acute disease may present with diarrhoea and colicky
abdominal pain and may be confused with appendicitis. Crohns disease is
associated with perianal sepsis.

YOUR ANSWER WAS CORRECT

Scenario 2

An 80-year-old man presented to his general practitioner (GP) complaining of sudden


onset of left-sided abdominal pain. He had recently suffered from a transient ischaemic
attack from which he had made a complete recovery.

B - Ischaemic bowel CORRECT ANSWER


Ischaemic bowel may arise either secondary to a cause of intestinal obstruction
or as a primary condition due to interruption of the arterial or venous blood
supply. A recent transient ischaemic attack suggests underlying vascular
disease.

YOUR ANSWER WAS CORRECT

Scenario 3

A 32-year-old pregnant lady presented to her GP with a history of vague abdominal


pain associated with nausea and loss of appetite. She had a long history of depressive
illness to which these symptoms had been attributed. She had recently noticed marked
reddish discoloration of her urine.

C - Porphyria CORRECT ANSWER

The acute porphyryas, inherited disorders of porphyrin metabolism, affect


women more commonly than men (5:1). Acute attacks are precipitated by many
factors including the cyclical hormonal changes around menstruation.
Associated mood disturbances similar to depression often result in referral to a
psychiatrist for antidepressant therapy. Discoloration of the urine especially if
left in sunlight is suggestive of this diagnosis.

YOUR ANSWER WAS CORRECT

45

Theme: Surgical jaundice


A Gallstone in the common bile duct

B Cholangitis

C Infective hepatitis

D Carcinoma of the head of the pancreas

E Mirizzis syndrome

F Mucocoele of the gallbladder

For each of the statements below select the most likely diagnosis from the list
above. Each option may be used once, more than once or not at all.

Scenario 1

A 35-year-old women presents with sudden onset of jaundice, fever and rigors and
severe pain. She is tender in the right upper quadrant but the gallbladder is not
palpable. Liver function tests (LFTs) show grossly raised bilirubin, AST (aspartate
transaminase) and alkaline phosphatase.

B - Cholangitis CORRECT ANSWER

The triad of jaundice, fever and severe pain is indicative of cholangitis. It is seen
more commonly in the Far East.

YOUR ANSWER WAS CORRECT

Scenario 2

A 76-year-old man presents with insidious onset of jaundice with some weight loss. On
examination the gallbladder is palpable in the right upper quadrant. Liver function tests
show a raised bilirubin, a grossly raised alkaline phosphatase and a mildly raised AST.

D - Carcinoma of the head of the pancreas CORRECT ANSWER


Courvoisiers law states that if in the presence of jaundice the gallbladder is
palpable then the jaundice is unlikely to be due to a stone.

YOUR ANSWER WAS CORRECT

Scenario 3

A 35-year-old woman on routine medical examination and biochemical screening is


found to have a mildly raised bilirubin and alkaline phosphatase. She has had one
episode of right hypochondrial pain following a cheese and wine party the previous
Christmas. Ultrasound examination showed the presence of multiple small stones in
the gallbladder, and a common bile duct with a diameter just over the upper normal
limit, but no abnormalities.

E - Mirizzis syndrome YOUR ANSWER

A - Gallstone in the common bile duct CORRECT ANSWER

If the obstruction is due to a stone the gallbladder is likely to be thick and


fibrotic and does not distend. The final patient has an asymptomatic stone in the
common bile duct (CBD), and if laparoscopic cholecystectomy is undertaken,
there is a high chance of finding a stone in the CBD on cholangiography.

YOUR ANSWER WAS INCORRECT

46

Theme: Abdominal pain investigations - diagnostic

A Upper GI endoscopy
B Serum amylase
C Erect chest X-ray (CXR)
D Supine abdominal X-ray (AXR)
E Angiography (mesenteric)
F Computed tomography (CT)
G Barium enema
H Barium meal - small bowel follow-through
I Abdominal ultrasound

For each of the patients described below, select the most appropriate
investigation from the list of options above. Each option may be used once,
more than once or not at all. You may believe that more than one diagnosis is
possible but you should choose the ONE most likely (diagnosis) diagnostic
investigation.

Scenario 1

An 80-year-old man presents to the emergency department with severe central


abdominal pain. He has recently had a myocardial infarction (MI) and he is in atrial
fibrillation. The abdomen is distended and tender. Rectal examination reveals an empty
rectum.

E - Angiography (mesenteric) CORRECT ANSWER

The most appropriate investigation for this clinical scenario would be a


mesenteric angiogram as the most likely diagnosis is a mesenteric artery
embolus (probably due to atrial fibrillation).

YOUR ANSWER WAS CORRECT

Scenario 2
A 21-year-old lady presents to the surgical outpatient clinic with a 2-year history of
abdominal discomfort and weight loss. She used to get central abdominal pain which,
in the past few weeks, has moved to the right iliac fossa. She says that she opens her
bowels 45 times a day and has noticed occasional blood and mucus in her stools.

G - Barium enema YOUR ANSWER

H - Barium meal - small bowel follow-through CORRECT ANSWER

The history of a young woman with long-standing lower abdominal pain which is
localising to the right iliac fossa, increased frequency of motions and stools
mixed with blood and/or mucus is suggestive of Crohns disease.A barium meal
with small bowel follow through would be the most appropriate investigation in
this patient.

YOUR ANSWER WAS INCORRECT

Scenario 3

A 49-year-old presents with severe epigastric pain and vomiting. On examination, her
pulse is 120/min, blood pressure is 124/82 mmHg and her respiratory rate is 20/min
(shallow respiration). Bowel sounds are absent. She suffers from rheumatoid arthritis
for which she takes regular gold injections, oral steroids and diclofenac. Her dose of
oral steroids was increased a week ago to treat exacerbation of chronic obstructive
airways disease.

C - Erect chest X-ray (CXR) CORRECT ANSWER

The patient in this clinical scenario has probably got a perforated


peptic/duodenal ulcer due to the intake of oral steroids and non-steriodal drugs.
An erect CXR would be the most appropriate investigation if a perforated viscus
is suspected.
YOUR ANSWER WAS CORRECT

Scenario 4

A 90-year-old lady is brought to the emergency department with a 7-h history of


copious fresh red rectal bleeding. She is tachycardic and hypotensive. Of note is a
longstanding history of rheumatoid arthritis for which the patient has been on NSAIDs.
Initial blood tests indicate disproportionately raised serum urea.

E - Angiography (mesenteric) YOUR ANSWER

A - Upper GI endoscopy CORRECT ANSWER

The presenting history and initial investigations allude to an upper GI bleed likely from
a peptic ulcer. One must remember that torrential upper GI bleeds may present as
fresh rectal bleeding instead of melaena (i.e. due to rapid gut transit). In such cases,
the initial investigation of choice, after adequate resuscitation, is upper GI endoscopy
to identify and treat any potential bleeding points.

YOUR ANSWER WAS INCORRECT

Scenario 5

A 75-year-old lady in the orthopaedic ward who is taking oral steroids for COAD, and
diclofenac for pain complains of sudden-onset acute epigastric pain. She is
tachypnoeic and tachycardic. Any movement exacerbates her abdominal pain.

C - Erect chest X-ray (CXR) CORRECT ANSWER

As in the third case scenario, this patient has most likely got a perforated
duodenal ulcer, and an erect CXR would be the most appropriate investigation.

YOUR ANSWER WAS CORRECT

Scenario 6
A 70-year-old lady with mental health problems presents to the emergency department
with peri-umbilical pain, gross abdominal distension, and absolute constipation.

D - Supine abdominal X-ray (AXR) CORRECT ANSWER

The most likely diagnosis in this patient would be a sigmoid volvulus and a
supine AXR is the investigation of choice. This would reveal an omega loop.

YOUR ANSWER WAS CORRECT

Scenario 7

A 34-year-old woman who was commenced on oral steroids recently for severe
asthma, presents with acute onset epigastric pain and vomiting. On examination, she
has decreased bowel sounds, guarding and rigidity.

C - Erect chest X-ray (CXR) CORRECT ANSWER

As in scenarios 3 and 5, this patient has probably got a perforated peptic ulcer
and an erect CXR would be most appropriate investigation. Air is seen under the
diaphragm in almost 90% of cases.

YOUR ANSWER WAS CORRECT

47

Theme: The acute abdomen


A Acute cholecystitis
B Acute pancreatitis
C Appendicitis
D Biliary colic
E Diverticulitis
F Gastroduodenitis
G Large bowel obstruction
H Leaking abdominal aortic aneurysm
I Meckels diverticulum
J Mesenteric infarction
K Ovarian cyst torsion
L Perforated duodenal ulcer
M Small bowel obstruction
N Ureteric colic
The following scenarios describe patients presenting with acute abdominal pain. From the
above list, choose the most likely cause. Each item may be used once, more than once, or not
all.
Scenario 1
A 78-year-old woman presents with a 4-h history of very severe constant central abdominal pain. She
looks very unwell and is in distress despite diamorphine. Examination reveals slight abdominal
distension with some central tenderness but no peritonism. She is afebrile, blood pressure 140/90
mmHg, pulse 95/min irregularly and respiratory rate 30 breaths/min. Blood gases pH 7.1, pA(O2) 11
kPa, pA(CO2) 4.6 kPa, HCO3- 18 mmol/litre, base excess -6.
J - Mesenteric infarction CORRECT ANSWER
J Mesenteric infarction
This womans irregular heart rhythm has provided the right environment for clot (think of
Virchows triad) and subsequent embolus formation. Emboli can lodge anywhere in the
systemic circulation including the mesenteric arteries. An acute embolus blocking the origin
of the superior mesenteric artery with propagating thrombus extending into smaller vessels
usually produces mesenteric ischaemia. It can be notoriously difficult to diagnose but should
be suspected especially when pain is out of proportion to evident clinical signs as in this
case. A full classification is:
Occlusive

mesenteric embolus, eg from the left atrium in atrial fibrillation

mesenteric arterial thrombosis secondary to atheroma

mesenteric venous occlusion, eg portal hypertension, and

Non-occlusive

occurs in patients with a grossly diminished cardiac output, eg following myocardial infarction.

YOUR ANSWER WAS CORRECT

Scenario 2
A 75-year-old man presents with a 12-h history of worsening left iliac fossa pain associated with an
exacerbation of his usual constipation. Assessment reveals a temperature of 38.2C and tenderness
and guarding in the left iliac fossa. White cell count 18.2 x 10 9/litre, haemoglobin 14.1 g/dl, C-reactive
protein 150.
E - Diverticulitis CORRECT ANSWER
E Diverticulitis
This is without doubt the commonest cause of acute left iliac fossa pain and peritonism in this
age group.
YOUR ANSWER WAS CORRECT

Scenario 3
A 39-year-old man presents with sudden-onset, right-sided colicky abdominal pain that he describes
as the worst hes had in his life. Abdominal examination is unremarkable.
N - Ureteric colic CORRECT ANSWER
N Ureteric colic
The history of sudden onset of severe colic indicates that you are dealing with a luminal
obstruction not an inflammatory cause. In a man of this age, ureteric colic is top of the list.
The diagnosis would be strongly supported by blood in the urine. An intravenous urogram or
computed tomographic study of the kidney and upper bladder can be used to confirm the
diagnosis.

YOUR ANSWER WAS CORRECT

48

Theme: The acute abdomen


A Acute cholecystitis

B Acute pancreatitis

C Appendicitis

D Biliary colic

E Diverticulitis

F Gastroduodenitis

G Large bowel obstruction

H Leaking abdominal aortic aneurysm

I Meckels diverticulum

J Mesenteric infarction

K Ovarian cyst torsion

L Perforated duodenal ulcer

M Small bowel obstruction

N Ureteric colic
The following scenarios describe patients presenting with acute abdominal pain.
From the above list, choose the most likely cause. Each item may be used once,
more than once, or not all.
Scenario 1
A 78-year-old woman presents with a 4-h history of very severe constant central
abdominal pain. She looks very unwell and is in distress despite diamorphine.
Examination reveals slight abdominal distension with some central tenderness but no
peritonism. She is afebrile, blood pressure 140/90 mmHg, pulse 95/min irregularly and
respiratory rate 30 breaths/min. Blood gases pH 7.1, pA(O2) 11 kPa, pA(CO2) 4.6 kPa,
HCO3- 18 mmol/litre, base excess -6.
J - Mesenteric infarction CORRECT ANSWER
J Mesenteric infarction
This womans irregular heart rhythm has provided the right environment for clot
(think of Virchows triad) and subsequent embolus formation. Emboli can lodge
anywhere in the systemic circulation including the mesenteric arteries. An acute
embolus blocking the origin of the superior mesenteric artery with propagating
thrombus extending into smaller vessels usually produces mesenteric
ischaemia. It can be notoriously difficult to diagnose but should be suspected
especially when pain is out of proportion to evident clinical signs as in this case.
A full classification is:
Occlusive

mesenteric embolus, eg from the left atrium in atrial fibrillation

mesenteric arterial thrombosis secondary to atheroma

mesenteric venous occlusion, eg portal hypertension, and

Non-occlusive
occurs in patients with a grossly diminished cardiac output, eg following
myocardial infarction.

YOUR ANSWER WAS CORRECT

Scenario 2
A 75-year-old man presents with a 12-h history of worsening left iliac fossa pain
associated with an exacerbation of his usual constipation. Assessment reveals a
temperature of 38.2C and tenderness and guarding in the left iliac fossa. White cell
count 18.2 x 109/litre, haemoglobin 14.1 g/dl, C-reactive protein 150.

E - Diverticulitis CORRECT ANSWER


E Diverticulitis
This is without doubt the commonest cause of acute left iliac fossa pain and
peritonism in this age group.

YOUR ANSWER WAS CORRECT

Scenario 3
A 39-year-old man presents with sudden-onset, right-sided colicky abdominal pain that
he describes as the worst hes had in his life. Abdominal examination is unremarkable.

N - Ureteric colic CORRECT ANSWER


N Ureteric colic
The history of sudden onset of severe colic indicates that you are dealing with a
luminal obstruction not an inflammatory cause. In a man of this age, ureteric
colic is top of the list. The diagnosis would be strongly supported by blood in
the urine. An intravenous urogram or computed tomographic study of the kidney
and upper bladder can be used to confirm the diagnosis.

YOUR ANSWER WAS CORRECT


49

Theme: Right iliac fossa pain (rare causes)


A Diverticulitis

B Fractured NOF

C Ileocaecal tuberculosis

D Leaking iliac aneurysm

E Lymphoma

F Meckels diverticulitis

G Psoas abscess

H Rectus sheath haematoma

I Ruptured corpus luteum cyst

J Strangulated spigelian hernia

K Tabes dorsalis

L Torsion of an undescended testis

The following are descriptions of patients with rare causes of right iliac fossa
pain. Please select the most appropriate diagnosis from the list. The items may
be used once, more than once, or not at all.
Scenario 1
A 23-year-old amateur footballer presents with a 12-h history of right iliac fossa pain.
The pain started while running and has become increasingly severe since. There is no
history of fever or vomiting. On examination he is in obvious discomfort and has a very
tender mass palpable low in the right iliac fossa.

H - Rectus sheath haematoma CORRECT ANSWER


H Rectus sheath haematoma
This occurs following rupture of the inferior epigastric artery, typically after
coughing or straining. The site of the haematoma is usually at the level of the
arcuate line and produces a mass in the relevant iliac fossa. The lump is related
to the muscles of the anterior abdominal wall, contraction of which makes the
lump indistinct. Bruising may not always be apparent. The condition occurs in
three distinct groups of individuals: elderly women, pregnant women and
athletic, muscular men. Exploration of the anterior abdominal wall, evacuation of
haematoma and ligation of the bleeding vessel may be required, however, small
haematomas can be treated conservatively.

YOUR ANSWER WAS CORRECT

Scenario 2
A 29-year-old Indian man presents with a 1-month history of pain in his right iliac fossa.
On direct questioning he says he has felt unwell over the past 6 months with a history
of malaise, anorexia and pain in his lumbar spine and right groin. On examination he is
thin and appears malnourished. Abdominal examination reveals the presence of vague
tenderness in the right iliac fossa and loin, in addition to a fluctuant lump in his groin on
the same side.

G - Psoas abscess CORRECT ANSWER


G Psoas abscess
The history of a long period of ill health and backache is highly suggestive of
spinal tuberculosis, the commonest site of skeletal disease. Caseation and cold
abscess formation extends into the paravertebral soft tissues with the formation
of a psoas abscess, which may point to the groin. There is a risk of cord
compression from vertebral collapse or pressure from the abscess. Treatment is
with anti-tuberculosis chemotherapy, with surgery reserved for drainage of
abscesses, skeletal deformity and cord compression.

YOUR ANSWER WAS CORREC


50

Theme: Pathologies of the femoro-inguinal region

A Psoas abscess

B Obturator hernia

C Femoral aneurysm

D Strangulated femoral hernia

E Enlargement of psoas bursa

F False aneurysm of the femoral artery

G Local abscess

H Strangulated inguinal hernia

I Infected sebaceous cyst

J Saphena varix

For each of the clinical scenarios listed below, select the lesion most likely to
occur in that scenario from the above list. Each option may be used once, more
than once, or not at all.
Scenario 1
A 60-year-old lady presents to the A&E with a temperature of 37.6C and vomiting of
48 hours duration. On examination, a localised swelling below and lateral to the right
pubic tubercle is noticed. The swelling is tender and irreducible with no cough impulse.

D - Strangulated femoral hernia CORRECT ANSWER


D Strangulated femoral hernia
Femoral hernia is due to the protrusion of extra-peritoneal tissue and some
abdominal contents through the femoral canal. It is more common in females
(two and a half times) as the inguinal ligament makes a wider angle with the
pubis. Femoral canal is bounded supero-anteriorly by the inguinal ligament,
infero-posteriorly by the pubic ramus and pectineus muscle, medially by lacunar
ligament and laterally by the femoral vein. Femoral hernia is differentiated from
inguinal hernia as it lies lateral and inferior to pubic tubercle, while the inguinal
hernia lies medial and above the pubic tubercle. Strangulated hernia is
characterised by pain, irreducibility, and absent cough impulse. Necrosis of the
involved viscus ensues with progressive loss of blood supply and this may lead
to local, and then systemic sepsis, if untreated. The patients may present with
general ill-health, vomiting and constipation. Associated pyrexia may be present
if the infection becomes systemic.

YOUR ANSWER WAS CORRECT

Scenario 2
A 46-year-old lady presents with a soft, non-tender swelling over the medial side of her
right thigh below and lateral to the pubic tubercle. The swelling has got a bluish tinge
and disappears on lying down. She is well systemically but has got bilateral varicose
veins.

J - Saphena varix CORRECT ANSWER


J Saphena varix
Saphena varix is a saccular enlargement of the vein, usually seen at the proximal
part of the long saphenous venous system. This is commonly seen in patients
with varicose veins. The patients present with a swelling over the medial part of
the upper thigh (where the long saphenous vein enters the femoral vein piercing
the cribriform fascia) and the swelling often has a blue tinge. It disappears on
lying down due to the emptying of the vein into the deeper system (femoral vein).
There might be a palpable thrill when the patient coughs and an impulse is felt
over the varix when the vein is tapped from below (Schwarts test).

YOUR ANSWER WAS CORRECT

Scenario 3
A 35-year-old man of Asian origin presents with a painless, fluctuant swelling over the
upper medial side of his left thigh. He has been previously treated for tuberculosis. His
ESR is 113. He has also got some tenderness over the lower spine and X-ray reveals
some opacity in this region.

A - Psoas abscess CORRECT ANSWER


A Psoas abscess
Psoas abscess is usually due to a cold abscess tracking down towards the
inguinal region from the spine along the psoas muscle. This is usually due to
tuberculosis affecting the spine (Potts disease). The abscess is usually
fluctuant, painless and not warm (hence the name cold abscess which is
characteristic of tuberculosis). Often there may be cross fluctuation (above the
inguinal ligament) due to an associated iliac abscess. The diagnosis is made
from the history and clinical examination. It can be confirmed either by
radiograph, ultrasound or CT scanning of the spine. Associated organ
involvement should be actively looked for such as lung involvement, for
example.

YOUR ANSWER WAS CORRECT

Scenario 4
A 28-year-old IV drug abuser presents with a mildly tender, pulsatile swelling over his
right femoral triangle. He gives a history of considerable bleeding from the wound
before the swelling developed. He is apyrexial and systemically well.

C - Femoral aneurysm YOUR ANSWER


F - False aneurysm of the femoral artery CORRECT ANSWER
F False aneurysm of the femoral artery
False aneurysm of the femoral artery occurs as a result of laceration or injury to
the arterial wall (usually tunica adventitia). The injury leads to local haematoma
which becomes contained in the surrounding normal tissue. There is often a
history of considerable primary haemorrhage from the wound which usually
settles with sustained pressure. This is common with IV drug abusers as they
may accidentally puncture the femoral artery when attempting to inject the drug
into the adjacent femoral vein as the artery and the vein lies in close proximity in
the femoral triangle. Distal pulsation in the affected limb is usually maintained.
There may be local bruit over the involved arterial region and the appearance of
the expansile pulsation in the region usually clinches the diagnosis.

YOUR ANSWER WAS INCORRECT

51

Theme: The acute abdomen: Investigation and management


A Appendicectomy

B Computed tomography scan

C Diagnostic laparoscopy

D Laparoscopic appendicectomy

E Laparotomy

F Regular clinical review

G Ultrasound scan

The following scenarios describe patients with various causes of acute


abdominal pain. Assuming that they have undergone resuscitation and basic
investigation, choose from the list above the most appropriate next step in
management. Each item may be used once, more than once, or not at all.
Scenario 1
A 70-year-old, hypertensive smoker with a 3-h history of sudden onset of worsening
lower back pain and bilateral lower limb paralysis.

B - Computed tomography scan CORRECT ANSWER


B Computed tomography (CT) scan
The clinical findings, taken together with this gentlemans associated risk
factors, strongly suggest a diagnosis of ruptured abdominal aneurysm.
The first step in investigation is a contrast CT scan to establish the diagnosis
and extent of the problem and so plan surgical or endoprosthetic management.

YOUR ANSWER WAS CORRECT

Scenario 2
An 80-year-old woman presents with a 24-h history of absolute constipation associated
with lower abdominal pain. On examination, she is grossly distended with localised
peritonism in the right iliac fossa. The plain film demonstrates large bowel obstruction.

E - Laparotomy CORRECT ANSWER


E Laparotomy
This woman is suffering from large bowel obstruction. The right iliac fossa
peritonism is secondary to caecal distension and wall ischaemia. Laparotomy
should not be delayed beyond resuscitation in this instance because caecal
perforation with faecal peritonitis is imminent (and usually fatal). Note that when
such signs are not present, further investigation to exclude pseudo-obstruction
or other pathologies should be performed (e.g. contrast CT).

YOUR ANSWER WAS CORRECT

Scenario 3
A 26-year-old woman presents with a 14-h history of right iliac fossa pain, associated
with nausea and vomiting. On examination there is tenderness and guarding in the
right iliac fossa. She reports, however, that her pain is similar to an episode she had 1
year ago when she torted an ovarian cyst and required surgery.

C - Diagnostic laparoscopy YOUR ANSWER


G - Ultrasound scan CORRECT ANSWER
G - Ultrasound scan
This young womans symptoms may be caused by an underlying appendicitis or
related to a recurrence of her ovarian cystic pathology. Ovarian cysts may tort
and infarct or rupture, producing severe abdominal pain and low-grade fever that
can closely mimic the signs and symptoms of appendicitis. an ultrasound would
be the next logical step in the management of this patient. It would enable
gynaecological pathology to be diagnosed or excluded in a number of patients
thereby directing the patient to the correct management plan (medical or
operative) and to the right team.

YOUR ANSWER WAS INCORRECT

Scenario 4
A 38-year-old man with a previous history of alcoholic pancreatitis presents with a 12-h
history of increasingly severe epigastric pain and vomiting. On examination, he is
dehydrated, tachycardic and has widespread abdominal peritonism. White cell count
16.2 x 109/litre, amylase 150. No free gas found on erect chest X-ray.

F - Regular clinical review YOUR ANSWER


B - Computed tomography scan CORRECT ANSWER
B Computed tomography (CT) scan
This is a challenging but not uncommon presentation. You cannot clinically
differentiate between acute on chronic pancreatitis with a normal amylase and a
perforated duodenal ulcer with no free gas (50% do not). Rather than performing
a laparotomy, a CT scan is appropriate, principally because of its high sensitivity
of picking up free gas/fluid.

YOUR ANSWER WAS INCORRECT


52

Theme: Management of groin hernias


A Bassini repair

B Herniogram

C Herniotomy

D Laparoscopic intraperitoneal repair

E Lichtenstein repair

F Low crural (Lockwood) approach

G High crural (Lothiessan) approach

H McEvedies approach (or modification)

I Shouldice procedure

J Totally extraperitoneal prosthetic repair (TEPP)

K Transabdominal preperitoneal prosthetic repair (TAPP)

L Truss

M Ultrasound scan

The following patients have all presented with groin hernias. Select the most
appropriate management option. Each option may be used once, more than
once, or not at all.
Scenario 1
An 80-year-old woman is admitted to hospital with a 20-h history of abdominal
distension, pain and vomiting. On examination, there is a small tender lump lying below
and lateral to the pubic tubercle with overlying erythema. The patient has been fully
resuscitated.

H - McEvedies approach (or modification) CORRECT ANSWER


H McEvedies approach (or modification)
The patient has an obstructed, strangulated femoral hernia. There are three
approaches to femoral hernia repair and although opinion varies, the safest
surgical approach (after resuscitation) in this patient (with the expectation of
bowel injury and resection) is a modified McEvedie using an incision resembling
half of a Pfannenstiel incision (the original McEvedie had a vertical incision
necessitating division of the inguinal ligament). The preperitoneal space is
tracked downwards and the hernia is opened. If a bowel resection is required a
laparotomy can easily be performed either through this incision or by its
extension to a full Pfannenstiel. If strangulation was not expected then the low
crural approach (basically a small incision over the lump) is preferred. The high
crural approach disrupts the posterior wall of the inguinal canal and has fallen
from favour (in some textbooks).

YOUR ANSWER WAS CORRECT

Scenario 2
A 56-year-old man presents at outpatients with a small reducible inguinal hernia. You
are a trainee of the Royal College of Surgeons of England and must perform the repair.

I - Shouldice procedure YOUR ANSWER


E - Lichtenstein repair CORRECT ANSWER
E Lichtenstein repair
Based on relative ease of procedure and low recurrence rate, this repair using a
polypropelene mesh is now recommended (including by the Royal College of
Surgeons) as the mainstay of primary hernia repair (replacing Shouldice which
was probably slightly more complex to perform adequately in most hands but
still has its proponents). The recurrence rate should be < 1%. Discussion
continues regarding laparoscopic approaches (listed). NICE recommends
laparoscopic hernia repair for recurrent and bilateral hernia. A randomised
controlled trial in the New England Journal (2004) suggests a higher recurrence
rate when used for primary hernia compared with current open approaches. The
Bassini repair is outdated (pain, recurrence etc).

YOUR ANSWER WAS INCORRECT

Scenario 3
A 32-year-old man presents to outpatients from the general practitioner complaining of
a small lump in the groin that comes and goes and causes pain when present. On
examination you find no evidence of hernia.

M - Ultrasound scan YOUR ANSWER


B - Herniogram CORRECT ANSWER
B Herniogram
This is the investigation of choice when no lump is evident. In contrast, if a lump
had been present but you were unconvinced that this was a hernia then an
ultrasound to distinguish the lump would be the preferred investigation.

YOUR ANSWER WAS INCORRECT

Scenario 4
A 2-year-old boy is brought in with a right groin lump. This proves reducible with
sedation, analgesia and cold packs. It is planned for him to return for day-case surgery
in 48 h.

C - Herniotomy CORRECT ANSWER


C Herniotomy
Herniorrhaphies, ie repair of the retaining wall, are not required for
infantile/childhood inguinal hernias where the cause is patency of the processus
vaginalis. The sac is identified and carefully dissected from the cord, its
contents are emptied and the sac is then ligated and excised. This should be
delayed (but only briefly) if the incarceration can be managed initially
conservatively as in this case.

YOUR ANSWER WAS CORRECT

53

Theme: Large bowel obstruction

A Caecal carcinoma

B Caecal volvulus

C Carcinoma

D Diverticular disease

E Faecal impaction

F Foreign body

G Intussusception

H Ischaemic stricture

I Pelvic metastases

J Pseudo-obstruction

K Sigmoid volvulus

The following patients have all presented with large bowel obstruction. Please
select the most appropriate diagnosis from the above list. The items may be
used once, more than once, or not at all.

Scenario 1
A 67-year-old woman presents with a 4-day history of colicky, central abdominal pain
and increasing distension. She gives a long history of a tendency towards constipation;
however, on this occasion she has not opened her bowels for 2 weeks, nor has she
passed any flatus. She describes episodes of left-sided abdominal pain over the
preceding 6 years. Her weight and appetite are unchanged. On examination she is
dehydrated and in obvious discomfort. Her abdomen is distended, tympanic to
percussion and a vague mass is palpable in the left iliac fossa. An empty rectum is
found on rectal examination.

E - Faecal impaction YOUR ANSWER

D - Diverticular disease CORRECT ANSWER

D Diverticular disease

The diagnosis is suggested by the long history of constipation and recurrent


abdominal pain in an otherwise fit and well elderly woman. Inflammation of one
or more diverticula may result in haemorrhage, perforation, or resolution with
fibrosis. Repeated episodes of inflammation can give rise to the formation of a
fibrotic, inflammatory stricture with symptoms and signs of bowel obstruction.
This may be acute, chronic, or acute on chronic. In many cases the clinical
picture can be difficult to differentiate from colonic carcinoma and a definitive
diagnosis may not be possible until histological examination of specimens
resected at laparotomy is done.

YOUR ANSWER WAS INCORRECT

Scenario 2

A 42-year-old man undergoes a posterior L4L5 spinal fixation. You are asked to
review him on the second post-operative day. He has gross distension but a soft non-
tender abdomen. He has not passed wind since the operation.
J - Pseudo-obstruction CORRECT ANSWER

J Pseudo-obstruction

In a large hospital, barely a week goes by without being referred such a patient
either by the medical, neurosurgical, or orthopaedic teams. Colonic pseudo-
obstruction probably represents about a quarter of all large bowel obstruction. It
is a reactive dilatation and ileus, presumed (although notproven) to be
secondary to disturbed autonomic (cholinergic) innervation. Classic conditions
that may precipitate it are sepsis, especially chest infections, cerebrovascular
accident/neurosurgery, fractured NOF, and spinal/retroperitoneal surgery (these
probably directly disrupt nerves indeed, the condition was first described in
this respect with retroperitoneal tumours (Ogilvies syndrome).

YOUR ANSWER WAS CORRECT

Scenario 3

A 60-year-old man is seen in the resuscitation area of The Emergency Department.


The nurse looking after him has moved him there because of his history of an
abdominal aortic aneurysm repair 14 weeks earlier. He gives a 7-day history of lower
abdominal pain associated with abdominal distension. He has not opened his bowels
over the past 3 days, nor has he passed any flatus. This is unusual as he has had
severe diarrhoea since his operation, which his general practitioner says is the result of
an infection he picked up while in hospital.

H - Ischaemic stricture CORRECT ANSWER

H Ischaemic stricture

Ischaemia of the left hemicolon is a recognised complication of abdominal aortic


aneurysm repair and occurs as a result of an insufficient collateral blood supply
from the marginal artery following ligation of the inferior mesenteric artery.
Presentation may be acute, with the passage of bloody diarrhoea, or insidious as
in the case described. The region of the splenic flexure is commonly affected
and chronic ischaemia may result in the formation of a stricture,
indistinguishable from a carcinoma on barium enema examination. Mesenteric
arteriography confirms the diagnosis.

YOUR ANSWER WAS CORRECT

The Answer

Comment on this Question


There are four common causes of large bowel obstruction in Western society:
1/carcinoma,

2/ diverticular disease,

3/ volvulus (principally sigmoid) and

4/ pseudo-obstruction.

54

Theme: Investigation and management of gastrointestinal bleeding


A Angiography

B Barium meal

C Colectomy

D Colonoscopy

E Double contrast barium enema

F Emergency underun of bleeding ulcer

G Flexible sigmoidoscopy
H OGD and adrenaline injection

I OGD and banding

J OGD and sclerotherapy

K Partial gastrectomy

L Proctosigmoidoscopy

M Red cell radionucleotide scanning

The above are all methods of investigating or treating gastrointestinal bleeding.


For the following scenarios please pick the most appropriate answer from the
list. Each item may be used once, more than once, or not at all. In these
scenarios it is taken for granted that all patients are appropriately resuscitated in
the first instance.
Scenario 1
A 57-year-old man presents with significant haematemesis and melaena. After
resuscitation he undergoes upper gastrointestinal endoscopy. This highlights an
actively bleeding duodenal ulcer with visible vessels at its base.

F - Emergency underun of bleeding ulcer YOUR ANSWER


H - OGD and adrenaline injection CORRECT ANSWER
H OGD and adrenaline injection
In such a situation it is recommended that haemostasis is attempted with injection of
1 : 10, 000 adrenaline. Although evidence-based practice has shown heat probe,
bipolar diathermy and injection sclerotherapy all to be effective, adrenaline injection is
often used in combination with these modalities, and available trials show that
adrenaline alone is equally effective. Repeated injection of fibrin glue or the application
of micro-clips should be limited to specialist centres where operators are undertaking
such procedures on a regular basis, and hence have special expertise. Only if this fails
or if there are other endoscopic or clinical indications should the patient undergo
surgery (under-running of ulcer).

YOUR ANSWER WAS INCORRECT


Scenario 2
A 30-year-old man presents to The Emergency Department with a short history of
bright red bleeding per rectum. On questioning he has a long history of noticing blood
on the paper and occasionally in the toilet pan. He is otherwise well; Haemoglobin is
13.7 g/dl.

D - Colonoscopy YOUR ANSWER


L - Proctosigmoidoscopy CORRECT ANSWER
L Proctosigmoidoscopy
This is the first investigation of choice and will yield the cause (piles) in many
cases, both in outpatients and The Emergency Department. If no evident cause
is found in the anorectum and fresh bleeding continues, a flexible
sigmoidoscopy would be the next step. In contrast, dark-red bleeding/melaena,
or associated change in bowel habit to loose stool/diarrhoea would favour a full
colonoscopy.

YOUR ANSWER WAS INCORRECT

Scenario 3
A 63-year-old woman presents with a history of melaena that is increasing in
frequency. Her last few motions have also contained fresh dark red blood; haemoglobin
10.9 g/dl. An upper gastrointestinal endoscopy is normal and an unprepared
colonoscopy cannot clearly identify a source of acute bleeding. Despite adequate fluid
resuscitation she continues to exhibit signs of hypovolaemia and a repeat haemoglobin
is 7.0 g/dl. A blood transfusion is commenced and although relatively stable, further
intervention is clearly necessary.

H - OGD and adrenaline injection YOUR ANSWER


A - Angiography CORRECT ANSWER
A Angiography
Despite aggressive resuscitation, this woman appears to have an unrelenting
small bowel/proximal colonic haemorrhage (melaena plus dark fresh blood).
Although ultimately she is likely to require a laparotomy, the intermediate step is
to undertake an emergency mesenteric angiogram in an attempt to discover the
origin of the bleed. This can localise bleeding in 5886% of patients who have
active bleeding (classified as a rate of 11.5 ml/min). Positive angiography may
allow immediate therapeutic intervention via embolisation with metal microcoils
or gelfoams. In addition, intra-arterial vasopressin at angiography has a good
initial haemostatic capability; however the rate of re-bleeding is high. Clearly if
these attempts fail (either to localise or control the haemorrhage) an emergency
laparotomy, with on-table endoscopy/enteroscopy, should be performed.

YOUR ANSWER WAS INCORRECT

55

Theme: Abdominal disease


A Appendix mass

B Caecal carcinoma

C Coeliac disease

D Crohns disease

E Intussusception

F Meckels diverticulum

G Mesenteric ischaemia

For each of the patients below, select the most likely diagnosis from the above
list. Each option may be used once, more than once, or not at all.
Scenario 1
An 80-year-old lady with atrial fibrillation has a 4-day history of abdominal pain and
some rectal bleeding.
G - Mesenteric ischaemia CORRECT ANSWER
G Mesenteric ischaemia
The finding of atrial fibrillation in a patient with sudden-onset abdominal pain
must raise the possibility of an embolic event. Embolism accounts for 2530% of
patients with mesenteric ischaemia. It is notoriously difficult to diagnose and the
passage of blood per rectum is relatively rare.

YOUR ANSWER WAS CORRECT

Scenario 2
A 16-year-old boy presents with a 6-month history of diarrhoea, vomiting and vague
right iliac fossa mass. He has a microcytic anaemia.

D - Crohns disease CORRECT ANSWER


D Crohns disease
A longer history of diarrhoea and vomiting over several months, together with a
vague right iliac fossa mass and established microcytic anaemia, is more
suggestive of Crohns disease because of iron loss from haemorrhage. A
macrocytic anaemia may also be seen as a result of vitamin B12malabsorption
due to terminal ileitis.

YOUR ANSWER WAS CORRECT

Scenario 3
An 18-year-old girl presents with a 4-day history of pain in the right iliac fossa, a
temperature of 38 C and diarrhoea.

A - Appendix mass CORRECT ANSWER


A Appendix mass
An appendix mass forms when an inflamed appendix perforates locally
and pus is prevented from spreading throughout the peritoneal cavity by
adherent omentum and small bowel. The patient has a history suggesting
appendicitis, and occasionally a tender mass may be felt in the right iliac fossa.
YOUR ANSWER WAS CORRECT

56

Theme: Inhalation of foreign bodies

A Left apical
B Right apical
C Right anterior
D Right anterior basal
E Right lateral basal

For each of the scenarios below consider which segment of the lung the foreign
body would come to lie in. Each option may be used once, more than once, or
not at all.

Scenario 1

A 12-year-old footballer is upright when he inhales a tooth.

D - Right anterior basal CORRECT ANSWER

YOUR ANSWER WAS CORRECT

Scenario 2

An 85-year-old man inhales his medication following a stroke.

D - Right anterior basal YOUR ANSWER

E - Right lateral basal CORRECT ANSWER


The reason here is purely postion. The child is stood up, the stroke patient will
either be slumped or lying down, which anatomically, makes one more likely
than the other.

The main bronchopulmonary segments are as follows:

For the right lung

superior lobe (1) apical, (2) posterior, (3) anterior

middle lobe (4) lateral, (5) medial

inferior lobe (6) superior (apical), (7) medial basal, (8) anterior basal, (9) lateral
basal, (10) posterior basal

For the left lung

superior lobe (1) apical, (2) posterior, (3) anterior, (4) superior lingular, (5) inferior
lingular

inferior lobe (6) superior (apical), (7) medial basal, (8) anterior basal, (9) lateral
basal, (10) posterior basal.

Since the right bronchus is the wider, and more direct, continuation of the
trachea, foreign bodies tend to enter the right bronchus, from then they usually
pass into the middle or lower lobe bronchi.

YOUR ANSWER WAS INCORRECT

57

Theme: Hepatomegaly
A Amyloid disease

B Biliary tract obstruction

C Cardiac failure

D Cirrhosis

E Haemochromatosis

F Hepatocellular carcinoma

G Hepatic vein obstruction (BuddChiari syndrome)

H Hydatid disease

I Infective hepatitis

J Leukaemia

K Liver abscess

L Lymphoma

M Metastatic carcinoma

N Polycystic disease

O Riedels lobe

The above are all causes of discrete or diffuse liver enlargement. For the
scenarios below please select the most appropriate diagnosis from the list. The
items may be used once, more than once, or not at all.
Scenario 1
A well 35-year-old woman is referred to you in the surgical outpatients by her
concerned general practitioner (GP). In his letter he documents an enlarged liver. On
examination, you notice a smooth mass extending from the costal margin towards the
right iliac fossa. Liver function tests requested by her GP are normal.

G - Hepatic vein obstruction (BuddChiari syndrome) YOUR ANSWER


O - Riedels lobe CORRECT ANSWER
O Riedels lobe
This scenario describes the congenital anomaly of a Riedels lobe. This is a
projection of normally functioning liver tissue downward from the right lobe of
the liver, below the costal margin and along the anterior axillary line. It can be
mistaken for a pathological enlargement of the liver or the gallbladder, but is a
normal anatomical variation (hence the unremarkable liver function tests).

YOUR ANSWER WAS INCORRECT

Scenario 2
A 58-year-old African man presents to The Emergency Department complaining of a 3-
week history of gnawing right upper quadrant pain, weight loss and abdominal
distension. His history reveals that he had a blood transfusion in his home country after
his involvement in a severe road traffic accident 10 years previously and hepititis.
Examination reveals jaundice, a nodular and enlarged liver, and the presence of
shifting dullness.

F - Hepatocellular carcinoma CORRECT ANSWER


F Hepatocellular carcinoma
Hepatocellular carcinoma is one of the most common cancers world-wide,
although it is rarer in the Western world. It accounts for 90% of primary hepatic
malignancies and is endemic in regions where hepatitis is rife. In this particular
case, it is probably associated with chronic hepatitis contracted from a
contaminated blood transfusion. It should be suspected in any patient in whom
liver function rapidly deteriorates on a background of known hepatitis or
cirrhosis.

YOUR ANSWER WAS CORRECT

58

Theme: Acute abdominal emergencies


A Acute mesenteric ischaemia
B Diverticulitis
C Appendicitis
D Mesenteric adenitis
E Ruptured abdominal aortic aneurysm
F Small bowel obstruction
G Sigmoid volvulus
For each of the cases below select the diagnosis from the list above that is most
likely to apply. Each option may be used once, more than once or not at all.
Scenario 1
A 64-year-old man with no past surgical history, but a murmur and an irregular pulse
presents with relatively sudden onset of severe diffuse abdominal pain and tenderness,
tachycardia and mild hypotension. Both femoral pulses are present, his white cell count
is 14 109/l and his blood gases show a metabolic acidosis.

A - Acute mesenteric ischaemia CORRECT ANSWER


Acute mesenteric ischaemia
A patient with a history of arrhythmia and sudden onset of diffuse abdominal
pain with an acidosis should be assumed to have acute mesenteric ischaemia.

YOUR ANSWER WAS CORRECT

Scenario 2
A 75-year-old woman with a history of constipation but no blood loss per rectum (PR),
presents with a 12 hour history of massive abdominal distension, constant pain, an
empty rectum, bowels not open for 2 days but no vomiting. There is little abdominal
tenderness or guarding.

G - Sigmoid volvulus CORRECT ANSWER


Sigmoid volvulus
Gross abdominal distension with little sign of obstruction and a history of
constipation in an elderly patient would raise the suspicion of a sigmoid
volvulus. A plain abdominal radiograph is usually diagnostic.

YOUR ANSWER WAS CORRECT

Scenario 3
A 10-year-old boy presents with a 12 hour history of lower abdominal pain, a low grade
pyrexia, anorexia and nausea but no vomiting. He has recently recovered from a minor
chest infection. On examination he is tender in both right and left iliac fossae, with
maximal tenderness in the periumbilical region, no guarding or rebound and some
cervical lymphadenopathy. The white cell count is 22 x 109/l and there are no pus cells
in the urine. Blood tests and urine dipstick are normal.

D - Mesenteric adenitis CORRECT ANSWER


Mesenteric adenitis
Right and left iliac fossa pain in a child with a recent history of illness, normal
blood tests and urine dipstick results, a normal or elevated white cell count, no
pus cells in the urine and cervical lymphadenopathy is likely to be due to
mesenteric adenitis. Early appendicitis cannot be excluded and the child should
be observed carefully either at home or in hospital.

YOUR ANSWER WAS CORRECT

59

Theme: Right iliac fossa mass

A Actinomycosis

B Appendix abscess
C Appendix mass

D Caecal carcinoma

E Crohn s disease

F Iliac artery aneurysm

G Iliac lymphadenopathy

H Ovarian carcinoma

I Ovarian cyst

J Pelvic kidney

K Psoas abscess

L Ruptured epigastric artery

M Tuberculous ileitis

N Tumour in undescended testis

The following are descriptions of patients with a right iliac fossa mass. Please
select the most appropriate diagnosis from the list. The items may be used once,
more than once, or not at all.

Scenario 1

An 18-year-old man presents to The Emergency Department with a 7-day history of


colicky right iliac fossa pain associated with a persistent fever and anorexia. On
examination, the patient has a temperature of 37.8C, pulse rate 98/min, and there is a
tender indistinct mass in the right iliac fossa.

C - Appendix mass CORRECT ANSWER


C Appendix mass

The natural history of untreated acute appendicitis is that it will resolve, become
gangrenous and perforate, or it will become surrounded by a mass of omentum
and small bowel that walls off the inflammatory process and prevents
inflammatory spread to the abdominal cavity yet delays resolution of the
condition. Such patients usually present with a longer history (a week or more)
of right lower quadrant abdominal pain. On examination the patient has a
persistent low-grade fever, mild tachycardia and there is a tender indistinct mass
in the right iliac fossa. The condition is usually best managed conservatively, as
the risk of perforation has passed and removal of the appendix can be difficult.
This differs from appendix abscess, when a perforated appendix becomes walled
off by omentum. Unlike an appendix mass, the patient with an appendix abscess
becomes systemically unwell with intermittent swinging pyrexia, rigors and
profuse sweating. Drainage, either under radiological control or surgically, is the
best initial treatment.

YOUR ANSWER WAS CORRECT

Scenario 2

A 31-year-old woman is referred to the outpatient clinic with a 1-year history of


recurrent episodes of pain in the right iliac fossa, associated with an increased
frequency of passing loose stool. On further questioning, she admits to a weight loss of
1 stone over the same time period. On examination there is a tender mass in the right
iliac fossa.

E - Crohns disease CORRECT ANSWER

E Crohns disease

The long-standing history of typical symptoms should alert you to this


diagnosis. Examination findings often only reveal right lower quadrant
tenderness; however, a mass can sometimes be felt secondary to thickened or
matted loops of inflamed bowel plus localised perforation with
fistulation/interloop abscess.

YOUR ANSWER WAS CORRECT

Scenario 3

A 67-year-old woman presents to clinic with anaemia, unexplained weight loss and
non-specific lower abdominal pain. On examination there is a distinct hard mobile
mass.

D - Caecal carcinoma CORRECT ANSWER

D Caecal carcinoma

This needs to be excluded in any patient presenting in their <st1:time ">sixth to


eighth decade with anaemia, unexplained weight loss, or abdominal pain.

YOUR ANSWER WAS CORRECT

The Answer

Comment on this Question


Note: The candidate can approach the differentiation of right iliac fossa masses
by remembering the few common causes or alternatively by taking a systematic
anatomical approach, starting at the anterior abdominal wall and moving back
towards the retroperitoneum (see right iliac fossa pain).

60

Theme: Upper gastrointestinal (GI) procedures


A Total gastrectomy
B Partial gastrectomy

C Oesophago-gastrectomy

D Excision of ulcer

E Repeat endoscopy

F Underunning ulcer

Select the most appropriate option from the list above for the treatment of the
patients below. Each option may be used once, more than once or not at all.
Scenario 1
A 79-year-old lady with a poor cardiac history and mild renal impairment is admitted
with haematemesis and melaena. A duodenal ulcer is found at endoscopy and is found
to be actively bleeding. The ulcer was injected with adrenaline; however 12 hours later
she was noted to have rebled with further passage of fresh melaena. She remains
haemodynamically stable.

F - Underunning ulcer YOUR ANSWER


E - Repeat endoscopy CORRECT ANSWER
New NICE guidelines (June 2012) suggest that in the case of post-endoscopic
rebleeding, patients should be offered repeat endoscopy. If this fails to achieve
haemostasis then surgery should be considered. If the re-bleeding patient is
hameodynamically unstable then interventional radiological procedures should be
considered (if available). In this case, the patient has significant comorbidities and so
the minimum safe procedure (in this case underrunning) should be performed.

YOUR ANSWER WAS INCORRECT

Scenario 2
A fit 59-year-old man with a known benign gastric ulcer on the greater curvature of the
stomach presents with melaena. He has completed an 8-week course of omeprazole.
He has required a transfusion of 6 units of blood and repeat endoscopy shows signs of
an active bleed.
B - Partial gastrectomy CORRECT ANSWER
This patients ulcer is known to be benign. Despite best medical therapy his ulcer has
not healed and so it should be excised. The available evidence is not conclusive, but
suggests that partial gastrectomy may have lower morbidity and mortality rates than
ulcer excision.

YOUR ANSWER WAS CORRECT

Scenario 3
An 85-year-old man is found at laparotomy to have a perforated gastric ulcer high on
the lesser curve. His past medical history includes COAD (chronic obstructive airways
disease) and angina.

D - Excision of ulcer CORRECT ANSWER


This patients age and significant co-morbidities suggest conservative surgery (shorter
anaesthetic, procedure time, lower burden of surgical trauma) is indicated. Also the
perforation is high on the lesser curve, excision of the ulcer could be performed safely
without injuring the gastric vasculature.

YOUR ANSWER WAS CORRECT

61

Theme: Investigation and management of gastrointestinal pathology


A Cervical exploration and drainage

B Colonic resection and primary anastomosis

C Conservative treatment (antibiotics, total parenteral nutrition,

nasogastric tube)

D Computed tomography scan +/- guided drainage

E Emergency gastrectomy
F Emergency thoracotomy

G Gastrograffin enema

H Gastrograffin swallow

I Hartmanns procedure

J Oesophageal resection

K Oesophageal stenting

L Omental patch repair

M Primary repair and drainage (oesophagus)

N Upper gastrointestinal endoscopy

The above are all methods of evaluating or managing gastrointestinal


perforation. For each of the following clinical presentations please choose the
most appropriate answer from the list. Each item may be used once, more than
once, or not at all.
Scenario 1
A previously fit 50-year-old man presents to The Emergency Department complaining
of retrosternal pain radiating to his back, on a background of 24 h of dysphagia. He
tells you that he underwent an upper gastrointestinal endoscopy for chronic dyspeptic
symptoms 3 days earlier. Apparently the procedure had been difficult, requiring multiple
attempts to intubate the oesophagus, before being abandoned. On examination he has
mild crepitus in the soft tissues around his neck and auscultation reveals a positive
Hammans sign. His temperature is 38.3C, blood pressure 130/75 mmHg, pulse
105/min. Water-soluble contrast swallow shows a small localised leak in the region of
the thoracic oesophagus.

C - Conservative treatment (antibiotics, total parenteral nutrition, nasogastric


tube) CORRECT ANSWER
C Conservative treatment (antibiotics, total parenteral nutrition, nasogastric
tube)
There remains significant contention about operative versus non-operative
management for oesophageal perforation. This is principally because surgical
intervention implies the need for thoracotomy, a major plus procedure with high
morbidity and lengthy recovery. There is general agreement that in the
conservative management of instrumental oesophageal perforations, selection
criteria should include small, localised leaks, with ready drainage back into the
oesophagus (following contrast radiography), no pleural contamination and low-
grade symptoms, with an absence of systemic signs of sepsis. As this man has
presented late (> 3 days), he has theoretically undergone a selection process of
sorts (ie if his perforation was more severe he would have deteriorated more
rapidly). Conservative management includes the aggressive use of
broadspectrum antibiotics, parenteral nutrition and nasogastric suction with
gastric acid suppression. Undoubtedly any clinical deterioration may cause
revision of the plan for conservative therapy.

YOUR ANSWER WAS CORRECT

Scenario 2
A 64-year-old woman presents to Casualty with a 24-h history of progressive left iliac
fossa pain. She has been conservatively treated for diverticulitis in the past (with prior
barium enema confirming sigmoid disease). On examination, she has localised
tenderness and guarding in the left iliac fossa. In addition, her temperature is 38.9C,
blood pressure 105/60 mmHg, pulse 110/min.

C - Conservative treatment (antibiotics, total parenteral nutrition, nasogastric


tube) YOUR ANSWER
D - Computed tomography scan +/- guided drainage CORRECT ANSWER
D Computed tomography (CT) scan abdomen +/-guided drainage
This woman is demonstrating signs and symptoms of septically complicated
diverticular disease (arising from micro- or macroperforation). The management
depends on the clinical scenario simple diverticulitis does not usually require
urgent radiological assessment and can be managed with bowel rest and
antibiotics. The more ill patient, as in this case, should undergo contrast CT
scanning to exclude free perforation and facilitate drainage of any abscess. The
patient with generalised peritonitis in general requires a laparotomy.

YOUR ANSWER WAS INCORRECT

62

Theme: Abdominal pathologies


A Carcinoma of the ileo-caecal region

B Carcinoma of the rectum

C Crohns disease

D Diverticular disease

E Endometriosis

F Mesenteric infarction

G Mittelschmerz

H Ruptured ectopic pregnancy

I Torsion of the ovary

J Ulcerative colitis

For each of the following situations, select the most appropriate cause of the
acute abdomen from the above list. Each option may be used once, more than
once, or not at all.
Scenario 1
A 73-year-old woman presents to the Emergency Department with lower abdominal
distension and pain relieved by passing flatus. She also gives a history of altered bowel
habit, a sense of incomplete evacuation, and blood mixed with her stools.

B - Carcinoma of the rectum CORRECT ANSWER


B Carcinoma of the rectum
Colonic malignancy is the second most common cause for cancer deaths in
the UK. Predisposing factors include neoplastic polyps, ulcerative colitis,
familial polyposis and a positive family history. Clinical presentation depends on
the site: classically left-sided colonic carcinoma presents with obstructive
symptoms, bleeding per rectum, altered bowel habit and tenesmus. In contrast,
right-sided tumours usually present with symptoms of anaemia from occult
bleeding. Sometimes the patient complains of pain in the right iliac fossa and
abdominal examination may reveal a mass over this region. In both cases, a
change in bowel habit is seen. These symptoms are not mutually exclusive and
in many cases there is considerable overlap.

YOUR ANSWER WAS CORRECT

Scenario 2
A 30-year-old smoker presents to the surgical outpatient clinic with a 4-month history of
diarrhoea and abdominal colic. The patient has lost weight and has recently noted
some mouth ulcers. A blood test reveals microcytic, hypochromic anaemia.

C - Crohns disease CORRECT ANSWER


C Crohns disease
Crohns disease is common in North America and Northern Europe. Unlike
ulcerative colitis, Crohns disease affects the whole of the GI tract (from mouth
to anus). Risk factors include a strong positive family history; smoking
increases the risk threefold. Acute Crohns disease may mimic acute
appendicitis. In chronic Crohns disease, there is mild diarrhoea, extending over
many months accompanied by intestinal colic; intermittent fever, secondary
anaemia and weight loss. With progression of the disease, adhesions, trans-
mural fissures, intra-abdominal abscesses and fistulas may develop.

YOUR ANSWER WAS CORRECT

Scenario 3
A 21-year-old lady is brought to the Emergency Department with severe generalised
lower abdominal pain. She is pale, tachycardic, and her blood pressure is 90/54
mmHg.

H - Ruptured ectopic pregnancy CORRECT ANSWER


H Ruptured ectopic pregnancy
Ectopic pregnancy occurs in less than 1% of pregnancies. The typical history of
ectopic pregnancy is one or two missed menstrual periods with other signs of
pregnancy (mastalgia, morning sickness and increased urinary frequency). In
ruptured ectopic pregnancy the abdominal pain is initially crampy, but
subsequently becomes a more persistent and generalised lower abdominal pain.
Irritation of the diaphragm leads to shoulder tip pain. There may be signs of
hypotension and hypovolaemic shock. Internal examination may reveal
guarding, rigidity and rebound tenderness. Frequently, altered blood may be
seen in the cervix and movement of the cervix produces abdominal discomfort.
In some instances a mass may be felt in one of the adnexae and the uterus is
frequently soft and bulky. Ruptured ectopic pregnancy warrants immediate
surgical intervention.

YOUR ANSWER WAS CORRECT

Scenario 4
An 83-year-old woman presents with a 12-hour history of severe generalised
abdominal pain associated with nausea and vomiting. Her blood pressure is 100/70
mmHg. She is in atrial fibrillation and her bowel sounds are absent.

F - Mesenteric infarction CORRECT ANSWER


F Mesenteric infarction
Although any of the three anterior abdominal aortic branches (coeliac, superior
and the inferior mesenteric vessels) may occlude, it is the occlusion of the
superior mesenteric artery (SMA) that is most commonly seen and causes
mesenteric infarction. Despite the presence of collateral vessels in the SMA, an
acute reduction in blood flow may not enable them to dilate sufficiently and
rapidly enough. Occlusion may be due to a thrombus or an embolus. Causes of
infarction include: atrial fibrillation, and more rarely, dissecting aneurysms and
vasculitis. Clinical features include persistent, severe and generalised abdominal
pain. Inflammatory markers may be elevated and arterial blood gases may reveal
a metabolic acidosis. This condition is a surgical emergency, as the patient
rapidly becomes toxic and may die from septic shock unless the infarcted bowel
is removed.

YOUR ANSWER WAS CORRECT

You might also like